quinta-feira, 20 de dezembro de 2018

ACERTO

Sempre vale a pena, quando requisitada, depositarmos nossa opinião sobre os assuntos mais complexos? Principalmente sobre políticas sociais, percebo que muitos não estão tão dispostos a entender o lado do outro, e outros tantos já nem tentam argumentar sobre isso. Nesse caso, silenciar-se é recuar?

Claro que vale e nem é pena nenhuma. É ótimo. Silenciar é recuar sim. É preciso sempre falar e argumentar contra preconceitos, opiniões equivocadas, posicionamentos incorretos e tudo de ruim que se apresentar. Sem medo de falar a verdade, doa a quem doer, inclusive a si mesmo, no caso da verdade prejudicar quem a diga. Se isso acontecer, que se sofra o prejuízo, mas não se negue a dizer a verdade. Não interessa se se fizer inimigos. Quem é inimigo da verdade é meu inimigo, seja quem for, até o meu pai.

Eu me considero de centro-esquerda, mas não consigo concordar com apoio que um parte da esquerda dá ao Regime do Maduro, Cuba, Correia do Norte. Já fui chamado de coxinha, direitista. A pessoa ser de esquerda é obrigada a defender esses regimes na sua opinião?

Claro que ser de esquerda não significa apoiar ditaduras. Eu sou de esquerda e rejeito terminantemente esses regimes, bem como o da antiga União Soviética, o da China e qualquer ditadura, seja de direita, seja de esquerda. A verdadeira esquerda não é, absolutamente, totalitária, pelo contrário, é democrática e, mais ainda, acrática. Na Rússia, os bolchevistas traíram a revolução e implantaram um regime despótico. Quem chama quem seja de esquerda e rejeite ditaduras de coxinha e direitista é uma pessoa totalmente ignorante a respeito do que seja uma verdadeira esquerda, além de ser uma pessoa desprovida de ética por defender um regime político despótico. É só contestá-la de frente e arrostar sua ignorância.

Ernesto fala sério! Sabemos que a política e a educação é uma desgraça neste país, me diga, por favor, como sobreviver ao ensino médio estando rodeado (a) de completos idiotas intolerantes, me diga como não ser apenas o que nos pedem: uma nota. Revolucionar está se tornando uma utopia? Tenho medo.

Claro que não é uma utopia e nem é difícil. Basta você ser o que quiser ser ser dar a menor importância para o que pensem e digam sobre você. Estude o que for preciso para tirar as notas e passar nos exames, mas não se contente só com isso. Vá além e estude tudo o que tiver interesse. Basta não ficar perdendo tempo vendo televisão e acessando o smartphone, nem dormir demais que você consegue facilmente. Não se importe em ser diferente e incomum. Isso não é anormal. Mas não dependa da aprovação de ninguém para ser como é. Seja você mesmo, mesmo que todos te rejeitem. Fique na sua e não ligue.

Quando afirma que a ONU deveria derrubar regimes autoritários pergunto: isto é função dela?? Penso que o papel da ONU seria o de aconselhamento e ajuda humanitária entre as nações. E de que forma a ONU derrubaria regimes: Promovendo guerras ou boicotes?

Não é, mas deveria ser. Porque respeitar regimes autoritários é desrespeitar a auto-determinação dos povos, já que esses regimes não são o que o povo governado por eles desejaria. O único regime legítimo é o democrático. Não há escapatória. Monarquias absolutistas, ditaduras e regimes totalitários são totalmente inadmissíveis num mundo justo. O consenso das nações tem que rejeitar esses regimes, mesmo que isso seja economicamente danoso. Então a ONU teria que fazer algo para derrubá-los. Não por meio de guerra, pois isso provocaria sofrimento no povo e nos soldados que atacassem essas nações. A solução é o total embargo diplomático e comercial, com o apoio aos grupos locais que pretendam derrubar tais regimes. É o caso da Arábia Saudita, China,Cuba, Vietnã, Venezuela, Coreia do Norte, Sudão, Irã, Síria, Zimbabue, Bielorrússia, Mianmar, Camboja, Egito, Omã e outros que, agora, não me ocorrem.

Pode-se aprender física superior apenas por diletantismo, sem fazer curso?

Isso é bem difícil, mas não impossível. A questão é que o estudo avançado de Física exige uma Matemática também muito avançada, difícil de ser estudada diletantemente. Não tem como escapar dessa Matemática. Quem diz que se pode entender Física apenas conceitualmente se engana. Não dá mesmo. Vá a uma livraria e dê uma folheada em um livro de "Métodos Matemáticos da Física", como o do Arfken, para ter uma ideia. Ou pegue um livro de Eletromagnetismo, como o do Jackson ou de Mecânica Quântica, como o do Cohen-Tannoudji ou, ainda, de Mecânica Clássica, como o do Symon.

Se eu jogar matéria (priodicamente) no Sol, logo, ele nunca vai perder massa, portanto ele nunca vai acabar?

Não é porque o Sol perde massa que ele, um dia, vai parar de funcionar e sim porque ele vai, gradativamente, transformar, por fusão nuclear, seu conteúdo de hidrogênio em hélio e, daí para a frente, nos elementos seguintes da tabela periódica, até o ferro. Chegando ao ferro, a fusão nuclear passa a ser endotérmica e não gera mais energia radiante, portanto ele não tem como conter a pressão da gravidade de suas camadas externas sobre as internas e vai se contrair com uma onda de choque que fará um ricochete e expulsará seu conteúdo mais externo para o espaço, deixando um núcleo remanescente que não gera mais energia e vai se esfriar lentamente até apagar. Isso é o que se chama uma estrela "anã branca". Quando ela esfriar passará a ser uma "anã marron" e, depois, uma "anã negra". Mas esse esfriamento dura bilhões de anos. As estrelas anãs não são gasosas e sim sólidas, numa estrutura cristalina. Se a massa do caroço remanescente for 1,44 (limite de Chandrasekhar) vezes a massa do Sol, a estrela se tornará uma estrela de nêutrons, que é um único núcleo atômico com a massa da estrela toda. Para caroços remanescentes de massa maior do que cerca de 2,7 massas solares (limite de Tolman-Oppenheimer-Volkoff), a estrela se transforma em um "buraco negro".

A democracia é o melhor regime para o Brasil?

O melhor é não ter regime nenhum, isto é, a acracia. Todavia, como ela não dá para ser instaurada antes que a humanidade evolua muito, a democracia resta como o único regime político aceitável seja em que lugar do mundo for. A ONU, inclusive, deveria agir sobre os estados não democráticos e derrubar seus regimes, instaurando neles uma democracia. Não consigo admitir que alguma nação não seja democrática, por enquanto.

Em tempo atuais, como se deve dar o estudo de crianças que possem habilidades especiais? Elas devem desenvolver só a área a respeito da qual possui pendência ou deve estudar todas as áreas de conhecimento? Deve estudar em escolas especiais?

Todo mundo deve estudar todas as áreas de conhecimento, para poder se inserir satisfatoriamente no mundo. Quem tenha aptidões especiais, então, deve se dedicar mais a elas, sem descurar do resto. Se for possível, deve estudar em escolas especiais sim, pois elas promovem os desafios necessários para o desenvolvimento das aptidões dos especiais. Só que isso nem sempre existe à disposição. Então tem-se que ficar com a escola comum mesmo e buscar desenvolver as aptidões especiais de forma pessoal, se possível com a compreensão e o apoio da família. Porque há muitos pais que desencorajam o desenvolvimento de aptidões especiais por considerarem algo incomum e, mesmo, elitista. Muitos não gostam de que seus filhos sejam diferentes do comum, mas os superdotados são diferentes mesmo e não há como não serem. O que é preciso é aceitar que ser diferente e incomum não significa ser anormal.

Professor, já li e reli várias vezes os livros do professor Pierluigi Piazzi e concordo com tudo que ele diz lá como o estudar POUCO mas TODO DIA. Só ás vezes que me dá uma falsa impressão de que o cara que está estudando para um concurso, Enem ou vestibular vai se dar bem melhor do que eu que estou(continuação)estudando POUCO mas TODO DIA. Mas eu acredito que seja apenas uma falsa impressão. Tenho também feito atividades que desenvolvem a inteligência, como: ler 1 livro por semana e também ginástica mental como resolução de exercícios lógicos. Abraço!

A questão maior é que o quanto o Pierluigi considera que seja "pouco", muitos consideram que seja "muito". Estudar "pouco" é estudar, juntamente com o tempo das aulas, umas oito horas por dia. Estudar "muito" é estudar uma dezesseis horas por dia. Esse muito não se revela eficaz, pois satura o cérebro que, então, não assimila mais nada.

Em sua opinião, pode a região central de nossa galáxia, a saber, uma fonte de rádio compacta chamada Sagitarius A, está afetando os sistemas vivos no Planeta Terra?

Não. A intensidade da radiação que vem de lá é muito tênue. As mutações que provocam a evolução dos seres vivos na Terra são devidas, principalmente, à radioatividade da própria Terra. Além do mais, radiação da faixa de rádio não é mutagênica. Uma explosão de supernova que houver nas cercanias sim, afetará a vida na Terra, pois emite muitos raios gama. Isso, contudo, é bem improvável, já que as estrelas vizinhas, de tamanho suficiente, não estão em estágio de se tornarem supernovas.

Quando a dor é necessária e quando ela é desnecessária?

A dor é necessária para avisar à pessoa que a sente que algo não está certo em seu corpo. Quem não sentisse dor poderia sofrer sérios danos por não saber que algo está lhe causando problema. Há casos, contudo, que a dor se torna tão grande que extrapola essa finalidade. Ou, então, quando se tem consciência de que se está sofrendo algum processo e se tem o controle dele, não seria necessário que se sentisse a dor. Talvez pudesse haver um mecanismo corpóreo de bloquear voluntariamente a sensação da dor, se se quisesse. Como, por exemplo, quando se está tratando de um dente.

Definiu-se critérios para decretar que uma sociedade é melhor ou pior que outra em diversos aspectos, logo estaria sendo etnocêntrico?

Não necessariamente. Poder-se-ia estar comparando duas sociedades de mesma etnia. Por exemplo franceses e belgas. Ou japoneses e coreanos.

Qual a razão dos Estados Unidos concentrarem tantos movimentos de supremacia branca?

Isso é uma herança histórica da escravidão, quando os brancos que colonizaram o país, vindos da Europa, consideraram que teriam o direito de possuir pessoas humanas trazidas à força da África para trabalharem para elas em regime de escravidão. A ideia de escravidão é muito antiga na humanidade, sendo aplicada por muitos povos sobre outros que eles dominavam. Não se tinha a concepção ética de que isso não pudesse ser aceito e de que todas as pessoas fossem iguais. Os que, até hoje, passados quase dois séculos da abolição da escravidão nos Estados Unidos, ainda acham que o fato de serem brancos lhes confere algum tipo de superioridade são pessoas que permanecem com esse tipo de conceito arraigado, isto é, de que existe quem seja superior e quem seja inferior. Mas isso acontece, também, por exemplo, com muitos japoneses, no Japão, em relação a outros povos, negros, brancos e, até, da raça deles, de outros países. É um problema de falta de evolução civilizatória de vários grupamento de pessoas. Há, também, mesmo em quem não aceite a escravidão, quem ache que mulheres, pobres, ignorantes, seguidores de outra religião, pertencentes a outras raças, homossexuais e outros diferentes, sejam pessoas inferiores. É falta de esclarecimento, que é uma junção da detenção de conhecimentos com a sabedoria de proceder a julgamentos éticos.

ainda não entendi como pode existir conceito de reta e ponto mas não definição. aquilo que você disse, da reta ser uma linha que se estende indefinidamente nos dois sentidos, não diz exatamente o que é uma reta?

Uma linha que se estenda indefinidamente nos dois sentidos não precisa ser reta. Pode ser uma sinuosa, por exemplo. Você poderia dizer que uma reta fosse uma linha que se estenda indefinidamente nos dois sentidos tendo sempre a mesma direção. Mas, se você buscar a definição de direção verá que é a propriedade comum de um feixe de retas paralelas, fazendo assim um círculo vicioso. Portanto não se consegue definir reta. Do mesmo modo que ponto é entendido como a entidade geométrica que não possui extensão e apenas localização. Isso, contudo, não define ponto, mas apenas dá uma ideia do que seja.

Professor, hoje vi várias pessoas de grupos anarquistas defenderem a agressão àqueles que se declaram nazistas, vi até o termo "legítima defesa". O que acha?

Agressão nunca é legitima defesa. Nazistas, fascistas, do mesmo modo que comunistas, socialistas, anarquistas, capitalistas, absolutistas, democratas, e por aí vai têm todo o direito de terem as suas concepções. O que não podem, sejam quais forem, é pretender impingir à força suas ideias nem cometer crimes em nome delas. Da mesma forma que os pertencentes às diferentes religiões e os ateus.

Com o aumento da fabricação de carros elétricos no mundo, como ficará a situação dos países exportadores de petróleo? E a situação da Petrobrás? Recentemente,no Brasil, se descobriu novos poços de pré-sal, cuja exploração encontra-se inviável devido ao baixo preço do petróleo no mundo.

A atividade petrolífera gradativamente se reduzirá à indústria petroquímica, sem o seu uso como combustível, como é o que tem que acontecer mesmo. Os produtores de petróleo deverão mudar suas atividades econômicas para outras e a Petrobrás ficará reduzida e uma indústria petroquímica. O mesmo vai acontecer com as usinas de produção de energia elétrica quando ela passar a ser produzida domesticamente em micro-reatores de fusão nuclear. Várias atividades econômicas do mundo já foram extintas ou drasticamente reduzidas. Isso é um processo normal do progresso. Uma atividade que também precisa ser extinta é a pecuária, tão logo a humanidade se conscientize da barbárie que é o holocausto dos animais para consumo como alimento, produção de couro e outros destinos.

Na matemática, há uma distinção clara entre definição e conceito? Tal distinção se faz através de uma definição?

Esta distinção não é matemática, é linguística. Conceito é a descrição do que algo venha a ser, pela enumeração de suas características e propriedades, que permitam identificá-lo e distingui-lo do resto. Definição, por sua vez, é o enunciado preciso do que alguma palavre se refira, ou seja, ao se definir, está-se dando nome a uma descrição, de modo que, ao se usar esse nome, simplesmente está se substituindo a descrição pelo nome que lhe representa, sem nenhuma ambiguidade. Note que, enquanto o conceito é algo " a posteriori", a definição é "a priori", isto é, completamente arbitrária e não advinda de nenhuma observação.

se a lua atrai a terra com a mesma intensidade que a terra atrai a lua, por que a terra não orbita em torno da lua?

Acontece que ambas orbitam um ponto entre elas que é o centro de massa do sistema. Esse ponto é tanto mais próximo da maior massa quanto a maior massa é maior do que a menor massa. No caso da Terra e da Lua, aquela é 81 vezes mais maciça do que esta. Sendo a distância entre elas, 385 mil quilômetros, o centro de massa estará a 4.695 quilômetros do centro da Terra, isto é, a 1.683 quilômetros abaixo da superfície da Terra, dando a impressão que só a Lua orbita a Terra e que a Terra não orbita a Lua.

Há casos em que a violência lhe é legítima?

Sim, para se defender de alguma tirania, contra os tiranos, ou para se defender de algum ataque bélico.

Deus como projeção da mente humana. É um argumento de Feuerbach. Mas, isso não séria uma falácia genética?

Eu diria que os conceitos dos deuses são invenções humanas, não correspondentes a entidades realmente existentes. Mas não é uma falácia genética. Pelo contrário, a genética predispõe os seres humanos a conceber tais tipos de entidades, uma vez que o funcionamento cerebral busca achar, para toda ocorrência, uma causa, haja visto que isso é contumaz na maior parte das que são acessíveis à observação. Então, quando ela não está patente, a humanidade recorreu a entidades invisíveis como agentes causadores de fenômenos, como tempestades, incêndios, terremotos, vulcões, furacões, doenças e várias catástrofes, bem como ocorrências fortuitas que beneficiassem as pessoas, sem que elas tenham feito nenhum esforço para tal. Daí a invenção dos conceitos dos deuses e, posteriormente, a instituição de organizações formais para regulamentar e promover o culto a tais entidades, as religiões.

Ernesto, por que, em mecânica clássica, a força gravitacional é diretamente ao produto das massas e não a soma (por ex.)? Por acaso, seria devido a cada diferencial de massa provocar uma curvatura no espaço-tempo, influenciando cada diferencial de um com todos os demais do outro corpo e n apenas um?

A razão de ser proporcional ao produto e não à soma é porque o produto é a operação matemática que contempla o fato de que a força gravitacional seja proporcional, simultaneamente, tanto à massa que exerce a força quanto à que sofre a força. Se qualquer uma delas for multiplicada por algum fator, a força fica multiplicada por esse mesmo fator, o que é constatado experimentalmente, por exemplo, na experiência de Cavendish. E o produto (mas isso a soma também) é comutativo, fazendo com que a massa que sofra também exerça, sobre a que exerce, uma força de mesma intensidade, de acordo com a lei da ação e reação. Todavia se uma das parcelas de uma soma for multiplicada por um fator, a soma não fica multiplicada por esse fator.

O fascismo nao se discute, se destroi?

Tudo se discute. Mesmo que seja para destruir. Como saber que é para ser destruído se não se discutir? Fascismo, nazismo, comunismo, socialismo, capitalismo, democracia, autocracia, anarquia, monarquia, república, presidencialismo, parlamentarismo, religiões, poligamia, homossexualidade, eutanásia, suicídio, qualquer tema polêmico tem que ser discutido, com toda a abertura para o levantamento de todas as opiniões.

Alguem ai ouviu falar no Efeito Mandela, acredita?

Sim. Mas supor que seja uma lembrança de fatos ocorridos em universos paralelos é totalmente despropositado pois universos paralelos, se por acaso existirem, não se comunicam com o nosso.

Entao, o homem foi a lua, mas nunca voltou la porque simplismente nao tem nada o que se fazer la.

Acontece que voltou. Várias vezes. E fizeram muita coisa. Depois de várias idas, deixaram de ir porque as viagens tripuladas eram muito dispendiosas e se poderia continuar as investigações por meio de artefatos não tripulados. Além do mais a necessidade de afirmação de superioridade tecnológica dos Estados Unidos em relação à União Soviética deixou de ser algo preciso de ser buscado.

Como você reage quando pessoas que seguem alguma religião tentam "pregar" para você?

Considero isso completamente legítimo e argumento, serenamente e com todo o embasamento, da impropriedade de suas crenças, ao tempo que busco convertê-las ao ateísmo, com toda educação e sem ofendê-las por sua crença. Faço isso com os Testemunhas de Jeová. Em geral as pessoas que pretendem me converter não conseguem e nem são convertidas por mim. Mas acabam me respeitando e agradecendo minha atenção para com elas. Sei que, em geral, elas querem o meu bem e, por isso, fico até agradecido por seu interesse por minha felicidade.

Você não tem nenhum livro guia, uma espécie de Bíblia, alguma obra filosófica ou um romance etc?

Nenhum em particular. Guio-me por mim mesmo, por tudo o que estabeleci como minhas convicções a partir de meus estudos e reflexões baseado em várias obras de vários autores. Concordo em parte mas não inteiramente com muitos deles. Até mesmo com a Bíblia, eu que sou ateu. Ou o Bhagavad Gita. Gosto muito do Bertrand Russell.

mas a bandeira da USA tinha movimento de ventania, como explica isso?

Não era de ventania e sim a oscilação advinda do movimento de colocá-la no lugar.

voce acha mesmo que o homem foi a lua? ou e uma montagem " filme " da nasa produzido pra enganar a humanidade?

Claro que foi. Não é uma questão de achar. Isso é completamente documentado. Se fosse uma montagem os soviéticos teria denunciado a fraude, pois eles acompanharam.

A POESIA PREVALECE

Mesmo que o céu se despedace,
que a galáxia se esfacele,
que o universo se rasgue,
a poesia prevalece!
Se nada mais restar de nós,
se o tempo cessar o seu curso,
se nada, enfim, mais houver,
a poesia prevalece!
Pois os sonhos engendrados
nas mentes encasteladas
em surtos de insanidade…
Os ideais almejados
de um mundo tão encantado,
só na poesia florescem.
Ernesto von Rückert

Por que você não se considera agnóstico?

Porque agnóstico é a concepção de que não se possa saber se deus existe ou não.
Acho que se pode saber sim e se sabe que não. Só que não se tem a certeza disso. Então sou um ateu cético e não um ateu agnóstico nem tampouco gnóstico (isto é, o que tem certeza de que deus não existe).

Ética é algo subjetivo?

De modo nenhum. Nem ética nem moral. A moral é consensual entre um grupo de pessoas e a ética é fruto da reflexão filosófica. A moral é relativa ao momento, ao lugar e ao estrato social da pessoa. Mas não é subjetiva. É objetiva, isto é, determinada pela coletividade. Às vezes pelo grupo dominante. E a ética resulta da análise filosófica das consequências das ações.

Algumas pessoas acham que o Temer deve ser mantido como Presidente,uma vez que vem "tentando acertar" e substituí-lo agora criaria um tumulto na nação porque resta pouco tempo de governo e o Rodrigo Maia,também, é suspeito de crimes (seria trocar seis por meia duzia. O que pensa a respeito?

Penso que a ética está acima de tudo. Se ele é corrupto tem que ser defenestrado, mesmo que isso atrapalhe a economia. E se o Maia também for, que também o seja. Até que sobre alguém que não seja. A necessidade de punir o crime nem é para dar exemplo, mesmo que dê. É que crime é algo inadmissível e tem que ser coibido, haja o que houver.

Qual seria a diferença entre Filosofia e Religião?

Enquanto Religião é um corpo de assertivas pretensamente comunicadas pela divindade (em verdade pelas pessoas que as fundaram), colocadas para serem atendidas sem questionamento, a Filosofia apresenta suas assertivas fundamentadas na razão e colocadas sob o crivo do exame, da refutação e da contestação. Inclusive quando cuidam do mesmo tema.

Ernesto, visto que existe um grande dilema entre religião e ciência, na sua opinião, porque as pessoas devem acreditar na ciência como forma absoluta de conhecimento?

Acontece que não devem, do mesmo modo que não devem acreditar nas explicações religiosas. Simplesmente porque a Ciência não tem as explicações definitivas a respeito da realidade. Toda explicação científica é provisória. Mas é o melhor que se pode ter e as pessoas devem acolhê-las por isso, estando, contudo, sempre alertadas de que não são absolutas. Além do que, há tópicos que a Ciência não é capaz de abordar, que, então, têm que ser abordados pela Filosofia.

Ernesto, o que resta válido do Positivismo ? o que foi superado ?

Do positivismo resta válida a rejeição de explicações teológicas para a realidade. Mas não é válida a rejeição de explicações filosóficas, nos casos em que as científicas não sejam capazes de abordar e sejam feitas de modo lógico e coerente, sem recurso a opiniões. Da mesma forma que se pode, sim, buscar a razão dos fenômenos, exceto a razão última.

Qual sua opinião sobre o Cristianismo?

Trata-se, como toda religião e crença religiosa, de uma completa enganação e, portanto, algo que não é bom para a humanidade. Tudo que as religiões possam fazer de bom, e elas o fazem, pode, perfeitamente, ser feito sem elas, como a caridade e a disciplina moral. Só que elas consomem muito tempo, dedicação, esforço e recursos das pessoas que poderiam ser carreados para a solução dos grandes problemas da humanidade. No caso do Cristianismo, em especial, ele se fundamenta no fato de que Jesus Cristo, com sua morte cruenta, teria redimido a humanidade, isto é, permitido que as almas dos justos mortos pudessem gozar da bem-aventurança celeste. Isso é uma extrema crueldade de Deus, que exigiu esse sacrifício de seu próprio filho em vez de, simplesmente, perdoar a humanidade. Não quero ser amigo desse cara. Mas, é claro, isso tudo é lorota, pois não existe deus nenhum.

Oi, sei que acredita que as pessoas são boas por natureza, mas isto não impediu a revolução francesa e cabeças rolarem, então... Queria saber se isto também te preocupa. Na faculdade, um professor meu muito engajado em gestão de negócio e programador falou deste cenário de desemprego como realidade:

Nunca disse que as pessoas sejam, necessariamente, boas por natureza e sim que as pessoas têm, por natureza, a capacidade de fazer, tanto o bem, quanto o mal. Só que há mais gente do bem do que do mal. E mesmo alguém que seja do bem, dependendo, pode fazer o mal, do mesmo modo que alguém do mal, dependendo, pode fazer o bem. O que a sociedade precisa é incentivar a prática do bem e coibir a prática do mal, criando condições para que o bem seja recompensado e o mal punido. É preciso que se estabeleça que, sempre, o crime não compense, de modo nenhum. E que todos tenham os mesmos direitos e oportunidades, de modo que jamais precisem se valer de meios escusos para levar a vida de modo digno, confortável, aprazível e feliz, para todos, sem exceção.

Perdoar é mesmo necessário?

Necessário não é. Mas é bom para que se alivie a consciência do peso de se estar com alguma restrição a outra pessoa. Reter o perdão deixa quem o faz com alguma angústia por isso, que não permite o gozo de uma plena felicidade.

Para você, o que é a felicidade?

Felicidade é um estado psicológico de quem esteja sentindo uma completa satisfação por estar vivendo em paz, alegria, conforto mental e prazer sensorial. Isso geralmente acontece com quem esteja sendo amado e não esteja sofrendo privações de alimento, abrigo, segurança, liberdade e que consiga a satisfação de seus desejos razoáveis. Um dos grandes fatores de felicidade é a constatação de que se está vivendo de modo significativo, ou seja, de que sua vida esteja sendo algo pelo qual o mundo fique melhor por ela estar ocorrendo do que ficaria se não estivesse ocorrendo.

Mestre, entre as áreas de radiodiagnóstico, radioterapia e medicina nuclear qual escolheria e pq?

Medicina, pois é muito mais abrangente e pode incluir as outras. Sendo-se médico pode-se fazer muito mais bem para a humanidade em termos de quantidade de pessoas beneficiadas por sua atuação.

O que pensa a respeito do financiamento público de campanha?

Em princípio, acho válido. Mas pode não vir a ser se os critérios forem de modo a onerar muito o tesouro nacional e terem uma distribuição viciada em favor de quem já detenha algum poder.

Os fótons podem ter quilômetros de comprimento?

Não há um consenso sobre qual seria o tamanho de um fóton. Todavia pode-se considerar que ele tenha o tamanho de seu comprimento de onda. Nesse caso, sim, poderia ter quilômetros, megâmetros ou gigâmetros de comprimento, bem como picômetros ou fentômetros.

Você concorda com o naturalismo metafísico?

Sim, concordo. É o meu entendimento, que eu chamo de "fisicalismo", ou seja, que a realidade objetiva, fora das mentes, é puramente natural, não existindo nada que seja sobrenatural, isto é, deuses e espíritos. Todavia a realidade objetiva natural não é apenas material. Os constituintes substanciais do Universo são campo, matéria e radiação. Além disso há os constituintes formais, como espaço, tempo, estruturas e ocorrências. Mas há realidades abstratas, isto é, existente apenas em mentes que as concebam, como conceitos, números, figuras, instituições, normas, valores e por aí vai. A Metafísica cuida, exatamente, dessas realidades e da categorização de todas as realidades, de suas características, de suas propriedades, de seus estados, das condições de sua existência e por aí vai. Também cuida de considerar a possibilidade ou não da existência de realidades sobrenaturais.

Pessoas inteligentes tem mas dificuldade em encontrar parceiros por terem um senso crítico mais aguçado?

Não sei disso não. O fato de serem mais seletivas não impede encontrar parceiros, dependendo do ambiente em que se situe. Certamente que não vai ser em barzinhos ou baladas. Quem sabe em Universidades, Bibliotecas, Museus e locais do tipo? Ou mesmo pela internet.

O que é o universo observável, como sabemos o tamanho dele, e o que a além dele?

Universo Observável é a parte dele que se consegue observar, pois a luz teve tempo de chegar de lá até aqui desde que ele existe. O que estiver além só poderá ser observado no futuro. Cada ponto do Universo tem o seu Universo Observável, que é uma esfera em torno dele com raio igual à distância que a luz pode percorrer desde que o Universo existe, ampliada pelo tanto que o Universo expandiu enquanto a luz fazia a sua viagem. Isso pode ser calculado. Veja estes artigos:

O que faz quando começa a ler um livro chato? Continua lendo assim mesmo ou desiste dele?

Depende. Se for um livro importante, cujo conhecimento vá preencher uma necessidade, eu o leio, mesmo que seja chato. Se não o for, não leio. Já fiz isso, por exemplo, com o "Ulisses", do James Joyce e como o "Arquipelago Gulag" do Soljenitsen.

As nossas preferencias são socialmente impostas na tua opinião ou não? Tem gente que tem preferencia por se relacionar com pessoas mais novas, outras mais velhas.

Claro que não. A sociedade promove uma grande pressão sobre as pessoas para que elas prefiram o que a maior parte prefere. Mas toda pessoa tem a liberdade de preferir o que quiser, mesmo em discordância do que a sociedade preconize. Portanto não há imposição, apenas pressão. Isso vale para qualquer assunto, não apenas a respeito das preferências em relação aos relacionamentos. Pode ser que, preferindo o que não seja o mais socialmente aceitável, a pessoa sofra algum tipo de represália. Todavia a pessoa pode superar isso e continuar a preferir o que prefira.

Os fotons viajam a velocidade da luz, assim nao possuem idade, certo? Pois, como os do inicio do Universo se transformaram em micro-ondas, nao podemos dizer que eles eram mais velhos?

A questão é que o tempo não passa para eles mesmos. Mas passa para os observadores externos. Assim eles envelhecem para os observadores externos, mas não para si mesmos. Eles têm frequências diferentes para os observadores externos em momentos diferentes mas, para eles mesmos, essas frequências ocorrem todas no mesmo momento.

Você costuma reler livros de literatura ou só os vê uma única vez?

Estou sempre lendo literatura. Geralmente um livro por mês. Dos outros três que leio no mês, normalmente, um é de Física ou Matemática, outro é de outra ciência ou Filosofia e o último é de divulgação científica. Todavia, em geral, não releio, pois prefiro pegar um que ainda não li. Mas, alguns, já reli, como todos os do Conan Doyle. Estou para reler os romances de Machado de Assis e de Eça de Queirós.

Boa noite! Na sua concepção, qual (o quê) foi o maior responsável pela formação do seu intelecto? Tipo, a pessoa que você se tornou hoje, quais são as raízes dela? Faça uma comparação com o Ernesto de 47 anos atrás.

O incentivo de meus pais desde minha tenra infância. O ambiente intelectual de minha casa. Os muitos livros que eu dispunha para ler. As conversas intelectuais, principalmente com meu pai. A curiosidade científica de minha mãe. Há 47 anos eu já era um intelectual. Só que tinha um cabedal menor de conhecimentos e de habilidades, pois isso eu venho aumentando a cada dia de minha vida. Da mesma forma que minhas constantes reflexões e meu costume de pensar muito sempre vão me levando a rever concepções que vou mudando ao longo da vida. Por exemplo já fui católico e hoje sou ateu. Já comi carne e hoje sou vegetariano, já fui de direita e hoje sou de esquerda, já fui arquista e hoje sou anarquista. Música Clássica eu sempre gostei desde bem criança, pois em minha casa se ouvia muito. Com o passar do tempo fui conhecendo cada vez mais compositores, obras e intérpretes. O mesmo eu digo de pintura, escultura, arquitetura, literatura. Ciências sempre foi o meu maior interesse, especialmente Física e Matemática. Mas as outras também. Minha curiosidade é imensa e isso é algo com que já nasci. Desde criança que eu ia na estação de trem de Barbacena para ver as locomotivas e entrava debaixo de caminhões para ver seus motores. Sempre queria saber tudo a respeito de tudo. E continuo assim, sem a menor vergonha de "pagar mico" agindo como criança para examinar tudo com grande curiosidade. Só não tenho curiosidade para saber da vida dos outros.

Na mídia quando aparece notícias de homens que abusam da enteada(o), sempre vem alguém que tenta culpar a mulher por ter se relacionado com o abusador, dizendo coisas do tipo: "a culpa é dela por ter se relacionado com alguém sem conhecer direito." O que você acha disso?

Isso é um total disparate. Não apenas entre padrasto e enteada ou entre madrasta e enteado, mas entre quaisquer pessoas. Qualquer abuso sexual é culpa de quem abusa e nunca de quem é abusado. Mesmo que a pessoa abusada possa ter ensejado tal abuso pelo fato de ter-se mostrado provocante, isso não tira a culpa de quem acedeu à provocação. A responsabilidade por se conter é sempre da pessoa que abusa. Não há desculpa por não se conter.

A ciência é neutra?

Em si, sim, no aspecto ético, isto é, ela não cuida de saber se aquilo que vá descobrir seja algo bom ou algo ruim, se favorece um ou outro. No entanto, como ela é feita por seres humanos, e estes são, necessariamente, éticos, os cientistas podem, e o fazem muitas vezes, ocultar alguma descoberta por ser algo que possa provocar o mal ou, até mesmo, fazer uso de alguma para propósitos malignos mesmo. Nesse caso não é a ciência que está sendo má e sim o uso que se possa fazer dela. A ciência deve descobrir a verdade sobre tudo, mesmo que isso possa ser usado para o mal. E, nesse mister, não é má. Mas se o que pode ser usado para o mal, assim o for, então esse uso é mau.

Professor, boa tarde. Você tem conhecimento de algum artigo explicando o fluído não-newtoniano?

https://en.wikipedia.org/wiki/Newtonian_fluid
https://en.wikipedia.org/wiki/Non-Newtonian_fluid
http://www.rheosense.com/applications/viscosity/newtonian-non-newtonian
https://www.sciencelearn.org.nz/resources/1502-non-newtonian-fluids
https://scholar.google.com.br/scholar?q=non+newtonian+fluid&hl=pt-BR&as_sdt=0&as_vis=1&oi=scholart&sa=X&ved=0ahUKEwj26cfb_8jVAhVMxCYKHebZCRMQgQMIJTAA

sexta-feira, 7 de dezembro de 2018

Por que o sofrimento existe?

O sofrimento advém de alguma interação de um organismo vivo senciente com o resto do mundo que provoque em seu sistema sensório ou cognitivo algum efeito não recompensador, como dor física ou emoções de medo, angústia, tristeza, isto é, algo que faça o sistema de recompensa ser contrariado. Isso acontece porque, simplesmente, as interações não se dão de acordo com o que seria desejável, mas de acordo com as circunstâncias que se apresentam, muitas das vezes de modo inteiramente aleatório. Também pode ser uma reação a alguma ação agressiva do sujeito que vá sofrer sobre algo externo, que pode ser, inclusive, outro ser vivo capaz de reagir a provocações ou, apenas, algum sistema inanimado, como uma parede sobre a qual se dê um murro. Não há nenhuma predestinação para que se sofra, nem se sofre porque se esteja pagando pelo mal que se fez em vidas pretéritas, uma vez que isso não existe, ou seja, a vida é só a presente e se encerra totalmente com a morte biológica. A afirmação de que "aqui se faz, aqui se paga", também é totalmente inverídica. Pode-se fazer muito mal e não se sofrer nada com isso. Como pode-se fazer muito bem e ter uma vida plena de sofrimentos. Não há correlação necessária entre o que se sofre e o que se faz.

O dia e a rotação da Terra

Muitos dizem que a Terra dá uma volta completa em torno de si em um dia. Não dá! Então de quanto gira? Vejamos. Enquanto a Terra gira em torno de si, ela também gira em torno do Sol e no mesmo sentido. Quando, pois, ela completa um giro em torno de si, não acaba no mesmo lugar em que começou, mas um pouco mais adiante. Isso significa que um ponto da Terra que estivesse exatamente abaixo do Sol a pino, no começo do giro, não estará mais ao fim do giro. Portanto ainda não se passou um dia. Quanto falta para isso acontecer? O tanto de giro a mais que a Terra tem que dar é exatamente a fração da órbita que ela executou nesse giro. Como um ano tem mais ou menos 365 dias e uma volta inteira tem 360 graus angulares, em cada dia, aproximadamente, a Terra caminha um grau em torno do Sol. Isso significa que ela tem que girar mais um grau em torno de si para voltar a estar apontada para o Sol. Ou seja, em um dia a Terra gira 361 graus em torno de si e não 360 graus. Quer dizer que, em um ano, ela dá uma volta inteira a mais do que o número de dias. Ou seja, para passar 365 dias, a Terra tem que dar 366 voltas em torno de si. Em consequência, uma rotação da Terra é um pouco menos do que um dia, ou seja, menos do que 24 horas. Quanto a menos? A 360ª parte de 24 horas, ou sejam, 4 minutos. Uma rotação da Terra leva, pois, 23h 56min e não 24 horas. Interessante, não?

Anos bissextos

Por que será que, a cada quatro anos, o mês de fevereiro tem um dia a mais? A razão é porque o tempo que a Terra leva para completar uma órbita em torno do Sol não é um múltiplo inteiro do dia solar (intervalo entre um meio dia e o próximo, no qual a Terra gira 361 graus em torno de si). Medindo-se cuidadosamente, acha-se que o ano tem 365 dias, 5 horas, 48 minutos e 48 segundos. Isso é mais ou menos 365 dias e um quarto de dia. Para não se considerar fração de dia, vê-se que, a cada quatro anos, fica faltando um dia aos 365 para se ter um ano. Então o ano tem 366 dias a cada quatro anos, escolhido como o dia 29 de fevereiro dos anos múltiplos de quatro. Mas… não é bem assim. Considerar esse excesso de 6 horas por ano significa contar 11 minutos e 12 segundos a mais do que seria preciso. Esse excesso se tornará igual a um dia inteiro, considerando que o dia tem 1440 minutos, a cada 128,6 anos. Para não considerar esse valor quebrado, basta considerar que esse número é, aproximadamente, 400/3, isto é, se, a cada 400 anos, não houver 3 anos bissextos, a conta ficará bem próxima da realidade. Então a regra fica sendo: Todo ano múltiplo de quatro é bissexto, exceto os que também forem múltiplos de cem mas não múltiplos de quatrocentos. Isso tira três a cada quatrocentos anos, os seja, os seculares não múltiplos de 400.

Órbita da Lua em torno do Sol

Como a Lua gira em torno da Terra enquanto a Terra gira em torno do Sol, no mesmo sentido de rotação, interessa saber como é a órbita da Lua em torno do Sol. Pode parecer que, nos lugares e momentos em que a Lua se mova em relação à Terra no sentido oposto ao que a Terra se move em relação ao Sol (na fase de Lua Nova), a Lua retrocederia em sua órbita, que formaria laços nessas posições. Mas não é assim. Como a velocidade orbital da Lua em relação à Terra é de 1 km/s e da Terra em relação ao Sol é de 30 km/s, na Lua Cheia, ela estará ultrapassando a Terra, por fora, com 31 km/s em relação ao Sol e na Lua Nova ela estará por dentro, com 29 kms, sendo ultrapassada pela Terra. Mas sempre girando em relação ao Sol no mesmo sentido, sem retrocesso. Com se fosse em uma pista de Fórmula Indy, ora a Lua ultrapassando a Terra( Lua Cheia), ora a Terra ultrapassando a Lua (Lua Nova), sempre a ultrapassagem feita pela direita. Outro aspecto interessante é que, mesmo na Lua Nova, quando a Lua está entre o Sol e a Terra, sendo ultrapassada por esta, a concavidade de sua órbita é voltada para o Sol, nunca para a Terra. A razão é que a força gravitacional sofrida pela Lua por parte do Sol é o dobro da que ela sofre por parte da Terra. Então a resultante, mesmo na Lua Nova, em que elas têm sentidos opostos, é voltada para o Sol, sendo a força centrípeta, que é sempre voltada para o centro da curva. Só que, nesse caso, o raio de curvatura sera 1,4 vezes maior do que a distância Lua-Sol e, na Lua Cheia, será 1,2 vezes menor. Ou seja, a órbita da Lua tem sempre a concavidade voltada para o Sol mas é ora mais encurvada (Lua Cheia), ora menos encurvada (Lua Nova) do que a órbita da Terra em torno do Sol. Note que a distância entre a Terra e a Lua é 400 vezes menor do que a distância entre a Terra e o Sol, de modo que, em um desenho delas, se a Lua estiver a 1 cm da Terra, o Sol estará a 4 metros.

PARALELAS E VERTICAIS

Um assunto mal entendido em geometria é o conceito de paralelismo. Duas linhas, retas ou não, para serem paralelas, não são, como se costuma dizer, linhas que não se interceptem apenas, mas que, além disso, mantenham uma distância constante. A distância de um ponto a uma curva é o comprimento do segmento de reta perpendicular à curva que passe pelo ponto considerado, indo do ponto à curva. Para que sejam paralelas, duas curvas têm que manter uma distância constante, tanto da primeira em relação à segunda quanto da segunda em relação à primeira. As curvas podem ser reversas, isto é, não jazerem em um plano. No caso de linhas retas, contudo, elas só serão paralelas se forem coplanares. Outro equívoco é dizer que duas verticais, lado a lado, sejam paralelas. Não são! Verticais são linhas retas cujas direções são indicadas pelo fio de prumo no local, ou seja, que tenham a direção do vetor peso. O peso, com relação à superfície da Terra, que não é um referencial inercial, é a resultante da força de gravidade com a força centrípeta (que é fictícia), devida à rotação da Terra. Sua direção aponta, aproximadamente (mas não exatamente, exceto para corpos nos pólos e no equador) para o centro da Terra. Assim as verticais são retas concorrentes e, portanto, não paralelas. Basta pensar nas verticais de dois lugares bem afastados, como Rio de Janeiro e Bagdad, para ver que não são paralelas. Contudo, mesmo duas verticais bem próximas, não o serão. Uma consequência disso é que não existe plano horizontal. Uma superfície é dita horizontal se ela for perpendicular às verticais em todos os seus pontos. Como as verticais não são paralelas, uma superfície horizontal não será nunca plana, pois um plano que seja perpendicular a uma vertical em um ponto não o será em outro ponto. As superfícies horizontais, mesmo de pequena área, são, aproximadamente, porções de calotas esféricas, cujo centro é o centro da Terra. Mesmo a superfície de uma piscina completamente sem ondas não é rigorosamente plana. Se a superfície for de uma lagoa onda haja correnteza, inclusive, ela não será horizontal, pois, para haver correnteza, é preciso que a pressão a jusante seja maior do que a vazante, o que só acontecerá se a altitude da superfície a jusante for maior do que a altitude a vazante. Mesmo que sejam poucos centímetros.

Você consegue fazer contas de divisão rapidamente de cabeça? Eu não consigo, sou burro?

Não necessariamente por isso. Pode até ser que seja. Mas tem que ver como você se comporta em relação ao aprendizado e à solução de outros tipos de problemas, inclusive não matemáticos. Fazer contas de cabeça é uma habilidade que se tem que treinar. Mesmo pessoas bem inteligentes, se não forem treinadas, não saberão fazer.

Me diga sua vida de solteiro ou namorando ou casado? Qual a sua preferencia? E Porque? Qual sua experiência?

O melhor e ser casado, mas não oficialmente, nem no civil nem no religioso. Com o compromisso apenas da palavra dada. Assim que tudo tem que ser. Sem contrato. A vida familiar é muito melhor, porque se tem amor, apoio, solidariedade, companheirismo, compartilhamento e sexo de forma estável e segura, com garantia. Mesmo que não seja com exclusividade e nem perenidade. Mas, não sendo com exclusividade, com ciência e consentimento dos envolvidos.

Uau! Incrível! Você leu todos ou alguns compra para guardar e depois, quando tiver tempo, ler?

Já li, pelo menos, uns três mil, ao longo de uns sessenta anos. Todavia não compro só para mim. Minha intensão é fundar uma ONG e disponibilizar minha biblioteca ao povo, de graça, mas sem empréstimo. O valor dela, se fosse para ser tudo comprado novo, agora, é de uns três meios de milhão de reais. Quero ver se acho alguém para doar uma sede para a ONG, que não valeria nem a quarta parte disso. Seria uma entidade sem estatuto, sem diretoria, sem contabilidade. Anarquista mesmo. Tudo de graça. Ninguém pagando nada e nem recebendo nada para trabalhar. Mas com compromisso e responsabilidade. Como tem que ser para a humanidade se tornar civilizada. Tudo feito por idealismo e não por dinheiro. Nem por reconhecimento, fama ou o que for.

Podia jurar que você defendia a ideia de que se deve gostar de algo antes de aprende-lo, mesmo que para isso você deva desenvolver o gosto por tal assunto, estou enganado?

Depende. Se é algo que não se precisa saber, então se deve aprender o que se gosta. Todavia há um mínimo, que é o que consta dos programas da Educação Básica (Ensinos Fundamental e Médio) que todo mundo tem que saber, goste-se ou não, senão não consegue se inserir na sociedade. Mas isso é bem pouco. Em verdade o Ensino Básico deixa de lado muita coisa e, também, aborda assuntos que não são necessários. Algo poderia ser tirado e substituído por temas relevantes, como saber cozinhar, costurar, plantar e cuidar das plantas, consertar aparelhos elétricos, mecânicos, instalações hidráulicas, fazer serviços de pedreiro, carpinteiro, marceneiro. Saber música, pintura, escultura, informática, administração, economia e temas assim. Também tem que ter educação cívica, moral, sexual (aprender a namorar e transar gostoso), nutricional, ecológica e outras do tipo.

Como achar a verdadeira vocação?

Primeiro vendo o que é que acha interessante. Então, experimentando mexer com o assunto e ver se, realmente, fica empolgado. Se não ficar, procurar outro. Se tiver mais de um assunto pelo qual se interesse, experimentar mexer com eles todos e ver qual mais gratifica. Porque o importante é trabalhar com o que se gosta, mesmo que não remunere tanto. Isso faz uma diferença enorme para a qualidade de vida da pessoa. Tanto quanto a escolha do cônjuge certo ou, talvez, até mais, porque é mais fácil mudar de cônjuge do que de profissão. As pessoas costumam não pensar em possibilidades extravagantes, como ser piloto de teste, comandante de navio, artista de teatro, marceneiro, escultor, maestro ou mesmo algo que nem requeira curso superior, como comerciante, fazendeiro, poeta, alfaiate e outras atividades. Geralmente só pensam em Medicina, Engenharia, Direito e coisas assim.

Schopenhauer é, em sua opinião, pessimista ou realista?

Pessimista.

Ué? Você concorda com a adoção de leituras em vestibulares? Eu sei que boa leitura pode até ser bom, mas de nada adianta se a pessoa não quiser ler, até porque ler é uma forma de lazer.

Adianta sim. Porque, se ela não quiser vai ser forçada e isso é muito importante. Pelo menos os poucos que são exigidos. Do mesmo modo que a pessoa tem que saber matemática, gramática, história, geografia, física, química e biologia, artes, mesmo que não goste. Isso é preciso, e até mais, para que a pessoa possa se inserir no mundo como alguém integrado à sociedade cultural e tecnologicamente avançada.

Você tem ideia de quantos livros há na sua estante? Há livros de quais gêneros ali?

Tenho uns sete mil livros. Um terço sobre Física e Matemática, principalmente Cosmologia, Relatividade, Gravitação, Física Quântica e Física de Partículas, em nível de Pós-Graduação. Mas tem também em nível de graduação e em nível médio. Outro terço de outras ciências, como História, Geografia, Química, Biologia, Geologia, Psicologia, Medicina, Engenharia, bem como Filosofia, Música, Artes Plásticas e divulgação científica e conhecimentos gerais. O último terço são livros de literatura: romances, contos, poesias, de autores de diversos países. O que tenho muito pouco são livros de Direito, Economia e Administração, bem como Agronomia. Também tenho uns quatro mil discos, a grande maioria de Música Clássica, além de fitas cassete. Umas dez mil revistas e jornais. Mais de quinhentos vídeos (incluindo Blu-rays, DVDs e fitas VHS). Partituras, Mapas, Gravuras, Programas de Computador e outros itens. Tenho nove estantes de três metros de largura, do chão ao teto, com sete prateleiras e duas camadas de livros, um atrás do outro, em todas elas.

Qual a importância dos cursos de humanas e sociais para a sociedade brasileira?

Para mim eles são importantes, sem dúvida, mas menos do que os referentes às ciências exatas, às biológicas e às administrativas. Acontece que o Brasil está saturado de pessoas com formação superior humanista e carente de engenheiros, administradores, agrônomos, enfermeiros e esse tipo de profissional. Geralmente a escolha de muitos recai nas ciências humanas em razão de terem um acesso mais fácil e, mesmo, de ser mais fácil se levar adiante o curso. Todavia humanistas de alto gabarito (o que a maioria dos formandos não é) também são necessários para o bem geral da sociedade. Como, do mesmo modo, artistas de todas as artes. E, principalmente, profissionais técnicos que não sejam de nível superior.

A crença em um Deus sumamente bom e na existência de um paraíso, séria um grande incentivo a um suicídio?

De modo nenhum, pois o suicídio, em todas as crenças religiosas, é um pecado, que impediria a alma da pessoa a ir para o paraíso.

Qual seria o melhor sistema político para o Brasil sair da crise o mais rápido possível? Você vê esperança para o Brasil?

O Parlamentarismo é o sistema que consegue contornar esse tipo de crise. Tenho esperança sim, mas não para breve, Acho que o conserto demora mais de um século. Se o povo insistir em não eleger essa cambada de corruptos e se muitos juízes agirem como o Sérgio Moro, não tendo nenhuma condescendência com poderosos, sejam políticos, sejam empresários. A começar nas pequenas cidades. Cadeia para essa corja toda. Em nível municipal, estadual e federal.

Professor, eu sofro por não conseguir fazer operações básicas da matemática, como: um simples conta de divisão, regra de três, qualquer um teorema. Eu já tentei tanto mas não consigo aprender. O senhor tem alguma dica para me dar?

Ou você sofre de discalculia e, então, tem que procurar um neuropsicólogo para se tratar, ou você experimentou, nos primeiros anos da escola, uma aversão à Matemática provocada por professoras ou professores mal preparados, que costumam apavorar as crianças, em vez de despertar a fascinação por ela. Nesse caso você tem que voltar atrás e recomeçar a estudar Matemática do principio, numa base lúdica, de modo a adquirir um deslumbramento por ela que lhe faça adorar seu estudo e, com isso, aprender. Talvez seja o caso de se valer do Método Kumon.

Por que o campo magnético da Terra aponta do Polo sul geográfico para o Polo norte, sendo o polo sul o polo norte magnético?

Porque se definiu o nome dos pólos de uma bússola em razão de qual extremidade apontava para o pólo norte e para o pólo sul, geográficos. Depois, quando se viu que, entre ímãs, os polos opostos é que se atraem, viu-se que o pólo magnético que fica perto do pólo geográfico norte, como atrai o pólo da bússola que foi chamado de norte, magneticamente, é um pólo sul.

Existem aplicações da análise combinatória na física? quais?

Claro que sim. A Física Estatística, por exemplo, que se fundamenta em probabilidades, faz uso direto de análise combinatória.

E o que adiantaria essas pessoas irem pra prisão? Só num Brasil surreal que isso aconteceria, não tem presídio nem pra criminoso quem atenta contra a vida. Vcs parecem mais lunaticos do que os próprios religiosos, isso me envergonha. Pior é o ateu concordar com uma causa cristã. Vocês são doidos.

Discordo inteiramente de você. Esses religiosos estelionatários e fanáticos são tão nocivos quanto os políticos, juízes e empresários corruptos. Têm que ser presos mesmo e não em celas especiais e sim junto com todos os presos nas celas comuns. Isso vale para ateus fanáticos também. Do mesmo modo que anarquistas, comunistas, fascistas e nazistas fanáticos. Fanáticos, pois ser comunista, anarquista, nazista e fascista, se não pretender impingir isso a ferro e fogo, mas pelo convencimento, é perfeitamente legítimo. Quanto a respeitar pessoas religiosas de bem, isso não significa concordar com suas concepções. Mas discordar delas e, mesmo, buscar convencer essas pessoas de seu equívoco, não significa considerar que elas sejam do mal.

O que faria se um "Testemunha de Jeová" batesse à sua porta? Você o convidaria para entrar e debateria sobre religião com ele?

Sim, inclusive já fiz isso várias vezes e, ao invés de ficar ouvindo a pregação deles, eu é que fiz minha pregação ateísta a eles e refutei tudo o que eles diziam. Educadamente, é claro e sem ofendê-los. No fim eles agradecera e foram embora.

Vi em uma resposta antiga sua que fala que gosta um pouco de Kant, especificamente que parte da filosofia dele?

Em sua concepção criticista, isto é, de não aceitar nem o empirismo nem o racionalismo, nem o realismo, nem o idealismo. Todavia discordo dele em sua concepção dos "juízos sintéticos a priori". Isso não existe.

As vezes recebe perguntas como "O que acha de tal frase" a ultima foi do nietzsche falando sobre Deus estar morto, mas é claramente uma metáfora e muio provavelmente sabe disso, então por que respondeu/responde como sendo algo literal?

Sei que é metafórica, mas uma metáfora tem que se reportar a algo verdadeiro. E essa não se reporta a nada que seja verdadeiro. Por outro lado, detesto metáforas e analogias. Para mim são uma forma de comunicação imprecisa, que dá margem a interpretações diferentes da que se pretende passar. Do mesmo modo que as ditas "indiretas". Não faço uso desses recursos. Sempre sou direto, franco, preciso. Elas podem ser admitidas em literatura e poesia. Mas em ciência, mesmo humana, e filosofia, de modo nenhum.

É possível ficar amigo(a) de alguém por quem estávamos apaixonados?

Claro! Não ficar, mas continuar, pois não se é apaixonado sem se ser amigo. Inclusive se o relacionamento amoroso se interromper, pode-se continuar amando, platonicamente, a pessoa e continuar a manter um relacionamento apenas de amizade. Tranquilamente. Só quando o relacionamento amoroso se desfaz em razão de algo de ruim que a outra pessoa tenha feito é que, geralmente, não se consegue continuar amigo. Contudo, se a outra pessoa passar a amar uma terceira, isso não é motivo para se cortar a relação amorosa. Se todos estiverem de acordo, é perfeitamente possível se manter um relacionamento triplo ou, até mesmo, quádruplo ou múltiplo, com todos se amando e sendo imensamente felizes, comprometidos, dedicados, cúmplices, sinceros, intensos, gratificantes, devotados e tudo o mais de bom.

Não sei quem é mais intolerante... O rapaz ser favorável ao holocausto por ser contra religiões monoteístas ou senhor, que acha que religiosos devem ser presos. É por isso que nós ateus nunca seremos levados a sério. Vocês nos envergonham, são tão idiotas quanto religiosos fervorosos, me desculpa!

Eu não disse que religiosos têm que ser presos. Somente os fanáticos que pregam a revolta contra outras religiões e os que cometem crimes em nome das religiões. Um religioso do bem tem que entender que outras pessoas podem ter outras religiões ou nenhuma e serem do bem também. Pelo contrário, mesmo sendo ateu, tenho grande respeito por pessoas religiosas sinceras e que fazem o bem. Mas não pelos estelionatários religiosos e nem pelos fanáticos e belicosos. Inclusive pelos ateus assim. Quanto a expressar sua opinião, você tem todo o direito e não precisa pedir desculpa.

Em relação a essa afirmação: "Todos os idiomas estão sempre evoluindo, logo o Português também, contudo, cada vez mais os alunos tem se mostrado folgados no aprendizado da língua Portuguesa, e isso vem dando resultados de uma evolução porca do nosso idioma." você acha que ela está certa?

Sim, acho. Acho que se precisa preservar um pouco a elegância do idioma.

Existe prova de que o tempo é uma dimensão? Essa é uma ideia necessária?

O que é uma dimensão? Trata-se de algo a que se pode atribuir um valor numérico e que esse valor se distribua ao longo de uma série de modo que parte do que venha a caracteriza qualquer coisa seja rotulada como possuindo um desses valores. Todo acontecimento se dá em algum lugar e em algum momento. Não há como algo acontecer sem que se possa considerar quando e onde aconteceu. Então a localização espacial e temporal são dimensões. No caso da espacial, é preciso se especificar três valores para se localizar algo em relação a um sistema de referência. No caso da localização temporal, basta um, isto é, quanto tempo antes ou depois do momento de um evento estabelecido como a origem da contagem dos tempos. Outros atributos não se configuram em dimensão, pois não é necessário que algo os possua para existir. Como temperatura, por exemplo. Somente sistemas materiais possuem temperatura. Um campo elétrico ou magnético, bem com uma radiação não possuem. O conceito de dimensão, incluindo o tempo, é necessário para especificar todo tipo de ocorrência do universo. Tudo acontece num momento e num lugar. Não há como se estar num que não é lugar e num que não é momento.

Triste, não? https://www.tecmundo.com.br/seguranca/120378-perdemos-internet-capitalistas-diz-cofundador-pirate-bay.htm

Extremamente. O que se pode fazer é, todo dia, botar a boca no trombone e minar, gota a gota, em tudo que for possível, o poder das companhias. Até que o capitalismo se torne completamente pulverizado de modo a que deixe de existir espontaneamente. Isso levará séculos ou, até, milênios. Mas é preciso agir no dia a dia durante essas centenas de milhares de dias.

"Deus está morto!" Você concorda com essa afirmação de Nietzsche?

Não, por duas razões. A primeira é que Deus, como entidade, não morreu porque, simplesmente, nunca existiu. A segunda é que o conceito de Deus como entidade existente, mesmo sendo inverídico, é, ainda, fortemente arraigado na humanidade, de modo que a fração que se libertou dessa crença equivocada, é mínima.

Talvez o professor não esteja tão atento aos seus seguidores, o que compreendo. Eles mesmos escrevem sobre esses problemas e depois se dizem indiferentes sobre suas dificuldades na vida social.

Não visito as páginas de quem me segue, exceto os que, realmente, dão respostas interessantes e valiosas para que eu, também, enriqueça meu cabedal de conhecimentos.

Qual sua opinião sobre os 'Illuminati'? Achas que é possível existir algo assim para a "dominação mundial" ou é só mais uma bestialidade inventada pelo homem? E por que cada vez mais aparecem essas teorias da conspiração?

É só uma bestialidade inventada. Pode até haver quem pretenda dominar o mundo, mas isso é impossível. Teorias da conspiração são elucubrações de pessoas paranoicas que o povo, em sua credulidade e falta de ceticismo, acata. Como acatam ensinamentos religiosos impingidos pela sociedade em que estão imersos sem crítica nenhuma, supondo serem verdadeiros por serem o que todos em derredor aceitam como tal. Nem pensam que há milhões de pessoas que aceitam como verdadeiras outras considerações inteiramente distintas, tanto é que existem várias religiões no mundo.

Quais os pré-requisitos de matemática e física são necessários para se entender bem tomografia computadorizada e ressonância magnética?

Para entender, realmente, como o aparelho funciona e como gera suas imagens, é preciso ter um conhecimento de Física e Matemática em nível de um curso de bacharelado, pois é algo bem complicado mesmo. Estou dizendo para ser capaz de projetar o aparelho e fazer o programa que gera suas imagens. Tem que ser um Engenheiro Físico.

Você tem muitos seguidores jovens e inteligentes, porém observo que boa parte deles está tão focada em considerada genial, que esquecem das outras áreas da vida. Boa parte é depressiva, virgem, anti social e finge não se importar em ter problemas. Talvez devesse mostrá-los que até o que é mundano ++ possui a sua importância.

Não percebi que quem me siga pretenda ser considerada genial. Acho que são pessoas com interesse em adquirir conhecimentos e esclarecer suas dúvidas. Isso não significa que se descurem do resto da vida. Intelectuais, filósofos, cientistas, humanistas, artistas e pessoas que lidam com atividades culturalmente envolventes, nem por isso são menos envolvidas em relacionamentos e na curtição de prazeres. O fato de ser virgem não é problema nenhum. Não ser extrovertida também não é problema. Ser introvertido não é ser depressivo e nem tímido. Mesmo os tímidos não são, necessariamente, deprimidos por isso. Pessoas assim não fingem não ter problemas. Elas não têm problemas mesmo. Alguma pode ser que tenha, mas não que sempre tenha. Principalmente se a pessoa possuir uma personalidade forte e não se importar com a opinião alheia sobre seu modo de ser.

Qual sua opinião sobre a adoção de leituras obrigatorias por vestibulares?

Acho muito bom. Assim se força a juventude a ler textos de qualidade ao mesmo tempo que se limita seu número para que não seja preciso ler tantos quanto seria preciso se não se estabelecesse sobre quais iriam cair questões.

O que acha disso: http://g1.globo.com/bahia/noticia/pre-vestibular-gratuito-voltado-para-negros-leva-mais-de-1500-alunos-para-a-universidade-sistema-opera-para-boicotar.ghtml

Acho muito válido. Mas a questão não é apenas em relação aos negros. Os brancos pobres também têm dificuldade de fazer curso superior por falta de preparo de base. O que é preciso é que a Educação Básica (Ensinos Fundamental e Médio) nas escolas públicas, seja, de fato, de qualidade, que permita às pessoas pobres competir de igual para igual, em termos de conhecimentos, habilidades e competências, com as pessoas ricas, sejam brancas ou negras. Acho que o racismo vai acabar quando não se mencionar, em nenhuma circunstância social ou profissional, a raça de uma pessoa, bem como o seu gênero e a sua condição econômica. Todo mundo tem que ter igualdade de direitos e oportunidades. Quanto aos deveres, quem for mais capaz e mais rido, tem que ter mais responsabilidades. Não que quem não seja não as tenha que ter também.

sou um canalha? ao debater sobre liberdade e religiosidade, outros debatentes adentraram no assunto do holocausto e, apesar de serem ateus, alegaram (hospedei a continuação) http://freetexthost.com/jpsr4ghknv

De fato as religiões são nocivas ao bem da humanidade. Mas isso não significa que se deva exterminar as pessoas que professam as religiões e sim convencê-las a deixarem de professar, tanto a sua quanto qualquer outra. Líderes religiosos fanáticos precisam ser presos por toda a vida, para não propagarem o fanatismo. Religiosos que cometem crime em nome da religião têm que ser presos também, não por serem religiosos, mas por serem criminosos. O holocausto foi uma barbárie sem par, igual às que fizeram Lênin, Stálin, Mao, Pinochet, Videla, os ditadores militares brasileiros, Pol Pot, Maduro e todos esse tiranos, inclusive Josué e Maomé, sejam religiosos, esquerdistas, direitistas ou o que mais possa ser. O crime não é justificado por nenhuma causa. Mesmo a do ateísmo. que não pode se valer dos métodos anti-éticos dos religiosos para fazer valer sua religião, como a inquisição ou a jihad.

E se você soubesse que sua família seria intolerante com o fato de ser ateu, ainda contaria?

Claro que sim. Do mesmo modo se eu fosse homossexual e eles não concordassem. Só que eu não sou e se fosse eles aceitariam, pois minha família é extremamente liberal, há várias gerações. Apesar de ter um modo de ser, até mesmo, aristocrático, minha família tem idéias muito avançadas. Meu avô paterno já era anarquista na virada do século 19 para o 20. Não acho que se possa fingir ser o que não se é para agradar ou não contrariar ninguém. Nem a família, nem a sociedade.

O que eu faço quando estou mentalmente cansado? Tenho que terminar um trabalho escolar hoje, só falta a formatação e algumas coisinhas aqui e ali, só que estou muito cansado, normalmente ouço música, mas isso até isso está me cansando.

Você tem duas alternativas. Ou faz o trabalho cansado mesmo e se esgota completamente ou desiste e fica sem fazer, assumindo as consequências.

A partir daí não concordei com a existência desse Deus ou de qualquer outro, mas o receio por tudo que me foi ensinado ainda existe. Sua família e amigos o entenderam quando se disse "abertamente" ateu?

Sim, bem como meus amigos e colegas. Isso porque eu sou um testemunho de que uma pessoa ateia pode ser uma pessoa honesta, justa, bondosa, prestativa, solidária, correta, ética e assim por diante. E porque eu não deprecio quem possua alguma religião, exceto aqueles que mencionei.

Algumas dúvidas minhas começaram justamente nisso; dizia ser evangélica sem concordar ou praticar a religião. Alguns anos atrás achei isso abominável e fui tentar entender a bíblia, acho que o pouco que entendi me fez ter certo receio de Deus e nunca mais tive a relação "Deus e serva" como antes. +

Recomendo que aprofunda bastante seus estudos, mas não se atenha apenas à Bíblia. Veja também as considerações de quem discorda das propostas religiosas, bem como as considerações das demais religiões não cristãs. E, principalmente, estude história e ciências, nos aspectos que tenham alguma relação com as religiões e com suas propostas de explicação da realidade.

O que você considera assédio? Ou entende como assédio?

Assédio é a abordagem de uma pessoa por outra que insista, mesmo que indiretamente, para que ela faça sexo consigo. Tanto faz ser uma mulher por parte de um homem, um homem por parte de uma mulher, uma mulher por parte de outra ou um homem por parte de outro. O assédio é agravado quando a pessoa que o faz se vale de uma posição superior, como em uma relação entre patrão(oa) e empregado(a), tio(a) e sobrinho(a) ou algo do tipo, caso em que a pessoa assediada possa sofrer alguma represália se não aceder ao assédio.

Ernesto, estou no último ano do Ensino Médio e pensando em cursar Física na universidade. Contudo, tenho muita dúvida em relação a qual Universidade pode me oferecer uma melhor graduação: USP ou Unicamp. Do seu ponto de vista, atualmente, qual delas oferece melhor suporte/visibilidade aos alunos?

Ambas são muito boas. Vai depender da área da Física que você for pretender trabalhar. Porque, mesmo na graduação, já é bom ir se enfronhando com o pessoal que mexe com a área. No meu caso eu prefereria a USP, pois há mais gente trabalhando com Relatividade Geral, Cosmologia, Astrofísica e Física de Partículas. Entre no site delas e veja quais suas áreas de pesquisa e quantos trabalham em cada uma. Em termos de qualidade de ensino, elas empatam como as melhores do Brasil.

Sempre que faço essa pergunta para ateus eles me dizem o mesmo que o senhor; que já foram religiosos... Foi difícil essa "transição" de alguém que tinha fé em um Deus como o da Bíblia mudar os seus pensamentos? Houve certo desconforto a respeito da salvação, céu e inferno, etc?

A razão é que estamos imersos em uma sociedade que tem profundos vínculos religiosos, no caso da nossa, cristãos e, em especial, no Brasil, católicos. Quando nasci, em 1949, então, ser ateu era uma imoralidade, quase um comportamento criminoso. Então a criação familiar, tanto minha quanto de quase todo brasileiro, era dentro do cristianismo. Mesmo o protestantismo era visto enviesadamente pela maior parte da população. O espiritismo, nem se diga. Atualmente, felizmente, já não há esse preconceito e existem crianças que já são criadas, desde o nascimento, em lares ateus. Ao se tornarem adultas, se não adotarem alguma religião ou crença, serão "ateus de nascença" (como, aliás, todo mundo é, pois, mesmo que seja criado desde criança em alguma religião, não nasce com nenhuma). A passagem do catolicismo para o ateísmo, no meu caso, não foi traumática, pois se fundamentou em alicerces bem estabelecidos, advindo de meus estudos e reflexões. Contudo não lamentei o fato de ter sido católico por algum tempo, pois, quando o era, era-o de forma convicta e sincera. Jamai fui um católico dito "não praticante", que apenas se diz católico e não vive segundo os preceitos do catolicismo. Isso, para mim, seja em relação à religião que for, é abominável. Se não se vai seguir os preceitos da alguma religião, não se diga que pertence a ela.

É possível resolver qualquer problema de física usando a análise dimensional?

Não. A análise dimensional é capaz de mostrar se uma solução seja necessariamente incorreta, mas não de que seja correta.

Mas eles te deram artigos ou só suas opiniões?

Eles, quem? A respeito de quê? São tantas as perguntas que chegam que é preciso que a pessoa faça referência ao que está dizendo para eu saber. De preferência dando o link da pergunta sobre a qual faz nova pergunta.

Estava observando seu Ask e o achei genial! Você nunca acreditou na existência de um Deus ou foi com certas observações que você percebeu que talvez sua linha de raciocínio fosse diferente das pessoas cristãs (ou de outras religiões)?

Já fui católico fervoroso e fiel. Como levo tudo muito a sério, resolvi estudar a fundo a doutrina católica e, comparativamente, as demais religiões. Ao mesmo tempo, como sempre também fui muito interessado em ciências e em história, fui estudando cosmologia, antropologia, evolução, neurociências e tudo o mais. Então cheguei a duas conclusões: primeira que a fé é um completo despropósito; segunda que o objeto da fé, isto é, a existência de uma realidade sobrenatural, é completamente ilusório. Então, entre meus 19 e uns 23 anos, primeiro deixei de ser católico e depois de ser cristão, ainda acreditando em um deus sob forma deísta. Depois me tornei agnóstico e, por fim, ateu cético. Desde então, meus estudos científicos, religiosos e históricos só têm confirmado o acerto de minhas convicções em não ser filiado a religião nenhuma e sequer manter qualquer crença religiosa sem ter religião. Todavia não menosprezo quem possua religião ou crença religiosa, desde que seja de modo sincero e que paute sua vida por suas crenças de modo ético. Mas desprezo visceralmente uma pessoa religiosa que não busque a santidade ou quem se valha da religião para extorquir o povo crédulo, como Edir Macedo, Valdemiro Santiago, Silas Malafaia e seus prepostos.

Tu já assistiu o filme donnie darko? Se sim, já leu teorias sobre o filme como exemplos físicos?

Não. Mas pesquisei a respeito e achei interessantíssimo, tanto que já encomendei o dvd e o livro. Depois que assistir e ler, comento.

professor você acredita em extraterrestres? A nasa está oferecendo dinheiro aquelas pessoas que se disponibilizarem a "proteger" a terra contra ataque extraterrestres, o que acha dessa história?

Acho que seres vivos extraterrestres e, mesmo, inteligentes, podem haver, uma vez que o Universo é muito grande. Isso, contudo, não é garantido. Todavia, mesmo que existam, não acho que nos visitem ou já tenham visitado. Acho interessante a leitura do livro "Sós no Universo?" de Peter D. Ward e Donald Brownlee.

Você realmente defende a liberdade religiosa? Aceitaria um centro de Umbanda e uma igreja de Satanás perto da sua casa como uma igreja Cristã?

Claro que sim, desde que não cometam crimes. Acontece que tudo o que as religiões, quaisquer que sejam elas, reverenciam como seres espirituais de uma realidade sobrenatural, simplesmente, não existem. Nem deuses, nem santos, nem anjos, nem demônios, nem satanás, nem orixás. Nada disso existe. Então as pessoas se reúnem em um templo religioso (o que, mesmo um terreiro de cultos afro-brasileiros são) para cultuar ficções. Se eles acreditam nisso e se o fazem com boa intenção, sem objetivo de fazer mal nenhum a ninguém, não tem problema nenhum. Mesmo o satanismo, como o de Lavey, não pretende fazer o mal. O problema é quando lideres religiosos, como muitos pastores neopentecostais e, mesmo, padres católicos, se valem da credulidade do povo para extorqui-lo financeiramente, num estelionato religioso que é, até, muito comum. Esses têm que ser condenados à prisão.

Qual comentário mais absurdo, no seu ponto de vista, você já ouviu envolvendo Deus?

Pessoas que sofrem desastres e sobrevivem, dizerem como Deus foi bom para com elas, não deixando-as morrer. Então foi ruim para com as que morreram? E que bondade se faz ao aleijar alguém, mesmo que não mate? Quando algo de bom acontece, dizem: "Graças a Deus", considerando que Deus é o responsável por tudo o que venha a ocorrer. Então, quando algo de ruim acontece, também têm que dizer "Graças a Deus". Mas, então, atribuem a desgraça a tudo, menos a Deus. Isso é incoerente.

Tenho um grande problema, a minha pessoa é indolente, preguiçosa, depressiva, negativista e introvertida. Não é o que eu queria ser, isso é fruto do mal trato, falta de amor, desprezo e várias outras coisas que sofri quando criança. Não tem cura. As vezes penso tirar a minha vida ou virar mendigo.

Não recomendo o suicídio, porque a vida é uma preciosidade ímpar e o fato de existir é uma raridade imensa. Portanto há que se curtir a vida. A introversão, contudo, não impede isso, se se compraz em si mesmo. Preguiça e indolência podem ser vencidas, do mesmo modo que a timidez, a vergonha, o medo e comportamentos similares. Basta se fazer o que se tem medo ou vergonha com medo ou vergonha mesmo que isso vai sendo superado. Desamor e desprezo, de fato, são deprimentes. Mas isso pode ser superado amando-se, mesmo sem ser amado. Sendo generoso, solidário, compassivo, prestativo, solícito. Tente pensar mais nos outros e no mundo do que em si mesmo. Aja. Vá fazendo coisas proveitosas para o bem do mundo, sem esperar retorno. Assim, se ele não vier, não se desaponta e se ele vier, se alegra.

Na sua infância você brincava com amigos ou era sozinho? Gostava de fazer o que?

Tanto brincava com amigos quanto brincava sozinho. Gostava de brincar com amigos mas gostava, também, de brincar sozinho. Nunca fui uma pessoa muito extrovertida e nem muito introvertida. Tímido eu nunca fui nem um pouquinho. Nunca fui mandão e nunca fui submisso. Sempre fui assertivo e franco. Porém com educação e modéstia. Mas nunca fui humilde. Como nunca fui soberbo. Não brigava, mas não fazia o que os outros queriam, se eu não quisesse, E nunca me importei com o que achassem de mim, bem como nunca pretendi ser igual a todo mundo. Sempre fui eu mesmo, com uma personalidade muito forte e bem diferente da maioria.

Quando adolescente, você já usou drogas ou bebeu, ou já te ofereceram isso, já foi tentado ao sexo vazio - aquele que é só por fazer -, enfim, você teve uma juventude com essas coisas?

Quando eu era adolescente, na década de sessenta do século vinte, isso não existia. Mas existia cigarro e bebidas. Só que eu nunca fumei e nem bebi (só poucas vezes e nunca a ponto de me embriagar). Quanto a sexo recreativo, também não fazia. Eu era da TFP e praticavamos a castidade. Só namorei depois que deixei a TFP, com 19 anos. E não se transava com a namorada.

Aprecia o compositor Elgar? Não me recordo de que tenha colocado alguma música dele no seu programa Primeira Classe.

Sim. Já coloquei algumas vezes, mas faz tempo. Gosto, especialmente, das variações "Enigma". Suas duas sinfonias e seus concertos para violino e violoncelo também são muito bonitos. Qualquer dia eu coloco no programa. Contudo ele foi um compositor que fez mais peças curtas. Suas marchas "Pompa e Circunstância" também são belíssimas.

Vi que você conhece o Robert Nozick e até deu uma sugestão de leitura da magnum opus dele. O Nozick, como um minarquista kantiano, considera que os homens têm direitos naturais por imperativo categórico, e diz que associações de controle nascerão na anarquia. O que acha dessas visões? Concorda?

Nozick não é a favor da anarquia e sim do estado mínimo. Eu já sou a favor da anarquia mesmo. Todavia considero que ela tenha que ser atingida por evolução e não revolução. Ou seja, ela será estabelecida no mundo quando a existência do estado, mesmo que mínimo e mesmo que mundial, não se faça necessária, justamente porque a humanidade será tão civilizada que ninguém buscará atingir os direitos de ninguém. Ou seja, todos serão "santos", numa concepção não religiosa de santidade, isto é, justos, bondosos, honestos, nobres, solidários, generosos, verdadeiros, enfim, detentores de todas as virtudes. Claro que o atingimento dessa situação demandará muitos séculos ou, até, milênios. Outro ponto em que discordo dele é que ele admite a propriedade privada como legítima. Não que eu ache que a propriedade de quem a tenha obtido de forma honesta deva ser confiscada, mas que a evolução da civilização levará ao compartilhamento da propriedade de tudo. Ou seja, as pessoas trabalharão com afinco e denodo, não para o proveito próprio, mas para o proveito de todos. Tudo de graça, inclusive sem troca. Tudo doado mesmo.

Já ouviu falar de William James Sidis? O que acha dele, e acredita que realmente tinha um QI de 300?

Sim. Acho que ele foi muito inteligente mesmo. Quanto ao valor de se QI, uma vez que não foi medido e sim estimado, penso que esteja exagerado. Suponho que pudesse ser uns 190. Não sei quais foram os parâmetros usados para essa estimação.

Acreditas em milagres?

De modo nenhum. Milagres seriam mágicas e mágicas não existem. Tudo o que acontece, acontece de acordo com o funcionamento da natureza. É impossível contrariá-lo. Ninguém é capaz disso. Muito menos nenhum deus, pois eles não existem.

Sobre o aquecimento global, recomendo ao sr. assistir essa série de vídeo do seu colega físico Alexandre Costa. Ele tem mestrado em clima e tem uma opinião diferente da sua. https://www.youtube.com/watch?v=KdzlCxU7x4c&t=533s

Vou ver. No entanto meu posicionamento não é um palpite e sim o resultado de minhas análises de informações colhidas de muitas fontes de gabarito. Cientistas mesmo, com doutorado e pós-doutorado em Meteorologia. Inclusive o diretor do Colégio em que trabalho, que é doutor em Meteorologia e ex-professor e ex-orientador de teses de pós-graduação na Universidade Federal de Viçosa.

O que leva um homem à sabedoria?

A reflexão e a prática da virtude. Todavia, há também, uma índole inata. Muitas pessoas são naturalmente sábias, bondosas, honestas, nobres, valorosas. Outra já nascem com predisposição para serem venais, corruptas, negligentes. Além do fator do ambiente familiar em que são criadas. É um conjunto complexo de fatores. Não há uma razão única que determine a sabedoria de uma pessoa.

Por que mesmo que com o maior índice de rejeição da história, segundo o fabuloso Ibope, não existe manifestações nas ruas contra Michel Temer?

Porque o povo está passado, inerte, abobalhado, descrente, pastel. E porque os líderes formadores de opinião e arregimentadores de massas não estão dispostos a isso. O problema é que o PT, que é contra o Temer, está desacreditado, porque também é corrupto. O Lula e a Dilma não têm moral para arrostar o Temer. Ninguém tem. Não há nenhum político de conduta ilibada que seja capaz de mobilizar a nação para uma cruzada ética de moralização da política e dos negócios. Nem os ministros do Supremo.

Você concorda que a Fundação Getúlio Vargas poderia escolher outro nome que melhor a representasse? (Me refiro ao ditador Getúlio Vargas)

Ah, sim. De fato não foi uma boa escolha. Só que ela foi fundada no governo dele.

Você acha Bakunin uma pessoa legal?

Não completamente, porque ele era a favor de revolução cruenta e, para mim, isso não é certo e nem eficaz. Mesmo que fosse eficaz, seria errado. Mas eu acho que suas propostas de estabelecimento da sociedade são muito boas. Todavia elas têm que ser atingidas por um processo educativo da humanidade, até que se torne, de fato, civilizada. Então não será mais necessário haver governo, polícia, forças armadas, judiciário, fronteiras, dinheiro, propriedade. Tudo poderá ser coletivo, com muito mais economia. Tudo mesmo. Inclusive maridos e mulheres. Todo mundo habitando Falanstérios. Ninguém rico e ninguém pobre.

Você concorda que a Fundação Getúlio Vargas poderia escolher outro nome que melhor a representasse?

Como não sou ligado à área de Administração e Economia, não sei se o Carlos Ivan Simonsen Leal é uma pessoa boa ou ruim para ser o diretor da Fundação Getúlio Vargas.

A medida que um corpo cai, sua energia potencial diminui, e sua energia cinética aumenta. Enquanto o corpo cai a soma das energias cinética e potencial permanece constantes. Achei isso num livro, parece contraditório, será que não entendi?

Não é nada contraditório. A energia potencial sempre aumenta quando se movem os corpos que interagem em sentido oposto ao que a força de interação os impele. Se a força for atrativa, como é a gravidade, a energia potencial gravitacional aumenta quando os corpos interagentes se afastam. No caso de um corpo próximo da Terra e a Terra, isto significa que ela aumenta quando a altura do corpo aumenta, em relação à superfície da Terra. Se se deixar que a força de gravidade atue, impulsionando o corpo para baixo, o valor do trabalho que ela realiza sobre o corpo é, justamente, o quanto de energia potencial vai se diminuindo com a queda (pois a definição do valor da energia potencial é o tanto de trabalho que se tem que fazer para vencer a gravidade). Ora, se não houver outras forças atuando (como a resistência do ar) a força de gravidade vai ser a força resultante e seu trabalho será o quanto de energia cinética está sendo fornecido ao corpo (pela definição do valor da energia cinética). Então, é claro, que o mesmo trabalho retira energia potencial gravitacional do corpo e fornece o mesmo tanto de energia cinética. O total das duas, portanto, permanece constante, pois o que se perde de uma, ganha-se de outra. Jogando-se o corpo para cima, é ao contrário: perde-se energia cinética e ganha-se energia potencial gravitacional.

https://ask.fm/wolfedler/answers/141716495645 Acredita que os números sorteados são aleatórios e, não escolhidos para ter alguém -ou não- especifico como ganhador. Por exemplo, ouço muito falarem que a mega-sena (ou quina, loto etc) é roubo, que escolhem um ganhador.....

Para mim isso é hoax. Não a loteria, mas isso que você está dizendo sobre ela.

Se antes de um raio cair em cima de mim, eu colocar um espelho na cabeça, o raio volta para cima?

Não. Porque o raio, como fenômeno meteorológico, não é um raio de luz, mesmo que emita luz. É uma descarga elétrica, constituída por elétrons e íons carregados em movimento com alta energia e violento fluxo. Ele é capaz de passar através do espelho, mesmo este sendo isolante elétrico (se for de vidro, se for metálico é condutor), já que atravessa o ar, que também é isolante.

O que permite a natureza a fazer coisas diferentes ao mesmo tempo?

Ora, o fato da natureza não ser uma entidade única e sim uma coleção de entes. Além do que, mesmo cada ente pode experimentar transformações distintas em cada aspecto de que ele é capaz de sofrer alteração. O oceano, por exemplo, ao mesmo tempo que sobe e desce, devido à atração do Sol e da Lua nas marés, também se encapela devido à ação do vento, se dilata devido à ação do calor do Sol e assim por diante. Cada fenômeno desses é experimentado pelo mesmo ente em razão de diferentes interações dele com os demais. Um ser vivo, então, sendo extremamente mais complexo do que um ser inanimado, é capaz de experimentar muito mais ocorrências distintas simultâneas. E um conjunto de seres, como uma floresta, uma nebulosa, um sistema planetário, um recife de corais e outros, mais ainda.

Interessante explicação sobre a diferença da não exigência em ser formado em Filosofia para filosofar dentre outras. Entretanto, um músico que não estuda Música, seria medíocre para compor ou tocar, razão pela qual, tiraria a Música deste rol que você elencou.

Não disse que não tenha que estudar música e sim que não tenha que fazer Curso de Música. Vários compositores, inclusive de música clássica, foram autodidatas, como Borodin, por exemplo.

Professor, lhe é aceitável o fim por tempo indefinido da democracia a fim de preservar a existência da sociedade?

De modo nenhum. Todo sistema político totalitário e autocrático é execrável por princípio e, portanto, inadmissível em qualquer circunstância. A democracia tem recursos para assegurar a segurança da sociedade e a permanência das instituições. A "Lava Jato" é um exemplo disso. Sempre que a democracia for defenestrada é preciso que a sociedade se erga e derrube o regime tirano que foi colocado em seu lugar. Mesmo que ele esteja fazendo algo de bom, mas, por não ser democrático, é necessariamente péssimo. A única alternativa aceitável à democracia é a acracia, isto é, a ausência total de governo. Mas isso só poderá acontecer quando a humanidade atingir a maturidade civilizatória, que permitirá a inexistência de qualquer tipo de crime, desonestidade, injustiça, negligência, preconceitos, intolerâncias, crueldades, malvadezas e esse tipo de coisas por total ausência de vontade de quem quer que seja de praticá-las.

Pra ser considerado filósofo basta ser formado em filosofia? É porque esses dias vi um tal de Pondé sendo denominado como filósofo mas nele, ao abrir a boca, não senti 1% de filosofia.

Ser formado em Filosofia não faz de uma pessoa um filósofo e sim um entendido em Filosofia ou um Professor de Filosofia, se tiver feito licenciatura. Para ser filósofo o que se tem que fazer é filosofar, seja formado em Filosofia ou não. Da mesma forma que um pintor ou escultor não precisa ter feito curso de Belas Artes, um escritor não precisa ter feito curso de Letras, um músico não precisa ter feito curso de Música, um político não precisa ter feito curso de Administração, um empresário não precisa ter feito curso de Economia, um fazendeiro não precisa ter feito curso de Agronomia e várias outras atividades. Mas um médico tem que ter feito curso de Medicina, um advogado tem que ter feito curso de Direito (além de passar no exame da Ordem dos Advogados), um engenheiro tem que ter feito curso de Engenharia e várias outras atividades regidas por lei. Filósofo, escritor, pintor, escultor, político, empresário e fazendeiro não são atividades regulamentadas (e nem devem ser). Músico é, mas quem souber fazer música sem ter curso de música, está feita e pronto. Claro que os cursos de Filosofia, Letras, Belas Artes, Música, Administração, Economia e Agronomia, exemplificados, fornecem um grande embasamento para se ser filósofo, escritor, pintor, escultor, músico, político, empresário ou fazendeiro. Mas, se não tiver talento para cada caso, não se será. E se tiver, pode-se o ser sem o curso.

Você disse que a velocidade de uma órbita rasante seria 37.000 km/h. Não seria Vr = Ve/2^¹/², onde Vr = Vel. rasante, e Ve = vel. de escape? Portanto Vr = 42,1 /2^!/² = 29.769km/h.

Realmente fiz a conta errada, mas o valor é dado por:
GMm/R² = mv²/R, donde v = (GM/R)^(1/2) = (6,67E-11 x 6E24/6,4E6)^(1/2) = 7,9E3m/s = 28.500 km/h.
A velocidade de escape é dada por GMm/R = mv'²/2, donde v' = v x 2^(1/2) = 40.300 km/h.

Professor, estou iniciando minhas leituras a respeito do anarquismo em geral. Recentemente vi um vídeo de uma palestra sua na UFV e sou estudante de História na mesma instituição, porém no curso temos uma leitura escassa a respeito do assunto. Poderia me indicar alguma bibliografia sobre?

Veja a coleção "Anarcobolsos" da Editora Hedra:
https://www.hedra.com.br/categoria/anarcos-de-bolso
Veja também o excelente livro:
"História das Idéias e Movimentos Anarquistas", de George Woodcock, em dois volumes.
Outras obras:
Textos Anarquistas - Bakunin - L&PM
Os Socialismos Utópicos - Jean-Christian Petitfils - Círculo do Livro
Utopias Anarquistas - Flávio Luizzetto - Brasiliense
Anarquia, Estado e Utopia - Robert Nozick - Martins Fontes
Evolucionary Socialism - Eduard Bernstein - Prism Key
What is Communist Anarchism? - Alexander Berkman - Dover (excelente)
The ABC of Anarchism - Alexander Berkman, Emma Goldman - Courier (ótimo)

assunto chato: você é a favor ou contra a abertura de uma investigação contra o nosso excelentíssimo Senhor Presidente Michel Temer?

Certamente que sim. Se ele está fazendo tudo para que ela não seja aberta é porque deve ter alguma culpa. Se ele fosse inocente de fato, então o que ele iria querer é que a investigação fosse feita mesmo. Essas manobras de compra de votos, inclusive, o incriminam. E os deputados que se vendem também são corruptos. É preciso que essa corja toda não seja reeleita. Mas o povo não tem vergonha na cara, como já dizia meu primo, Rui Barbosa.

Voce ja leu sobre o caso de Anneliese Michel? Se ela nao tivesse parado o tratamento medico e recorrido a religiao, certamente nao chegaria naquele estado profundo de esquizofrenia.

Sim. A substituição da medicina por tratamentos religiosos tem que ser considerada um crime. A condenação dos pais e dos exorcistas foi merecida.

É possível uma pessoa que não saiba fazer contas, como um portador de discalculia, ter sucesso acadêmico? Na área de humanas, obviamente.

Sim, perfeitamente. A pessoa com discalculia pode ser bem inteligente sob outros aspectos.

Acredita na mega-sena?

Como assim? Acreditar que ela existe? Claro que existe! Não é algo para se acreditar ou não e sim para se constatar. Acreditar em quê, a respeito dela?

Professor, qual é o mais indicado: dormir pouco ou dormir muito? Às vezes você diz que dormir muito consome tempo, já outras você diz que uma boa noite de sono ajuda na assimilação do conteúdo. Qual é o correto a se fazer, afinal?

Eu não disse que uma longa noite de sono seja necessária para a fixação do aprendizado e sim que uma noite de sono é necessária para a fixação do aprendizado. Mas não precisa ser longa. Cada um tem a sua necessidade de tempo de sono. Se dormir muito menos, terá problemas de saúde. Se dormir muito maís, terá problemas de saúde. À medida que se envelhece a necessidade de sono vai diminuindo.
http://odia.ig.com.br/noticia/mundoeciencia/2015-02-15/quantidade-ideal-de-sono-depende-da-faixa-etaria.html
http://veja.abril.com.br/saude/estudo-revela-horas-de-sono-necessarias-para-cada-idade/#
De fato, dormir é um desperdício de tempo. Porém é necessário. Então o caso é dormir o suficiente, mas não mais do que o suficiente.

O estudo da geometria analítica já é possível a partir do conhecimento dos axiomas da geometria euclidiana? Além da geometria analítica, é aconselhável o estudo da geometria vetorial?

Só com o conhecimento da geometria não se desenvolve a geometria analítica, pois ela é fundamentada na álgebra associada à geometria. Quanto a ser vetorial, a geometria analítica é vetorial mesmo. A adjetivação de analítica na geometria não tem o mesmo significado que essa adjetivação em mecânica. Além da geometria analítica é importante saber, também, a geometria diferencial, que é a associação da geometria com o cálculo diferencial. Ela requer o conhecimento da geometria analítica.

O estudo da geometria analítica já é possível a partir do conhecimento dos axiomas da geometria euclidiana? Além da geometria analítica, é aconselhável o estudo da geometria vetorial?

Só com o conhecimento da geometria não se desenvolve a geometria analítica, pois ela é fundamentada na álgebra associada à geometria. Quanto a ser vetorial, a geometria analítica é vetorial mesmo. A adjetivação de analítica na geometria não tem o mesmo significado que essa adjetivação em mecânica. Além da geometria analítica é importante saber, também, a geometria diferencial, que é a associação da geometria com o cálculo diferencial. Ela requer o conhecimento da geometria analítica.

Por que um corpo em órbita da Terra se move muito mais rápido do que um que está na superfície?

Um corpo em órbita é um corpo que foi jogado para o lado de certa altura, com tal velocidade que, mesmo que esteja sempre caindo, enquanto isso, vai andando para frente e, como a Terra é redonda, mesmo que ele busque o chão, o chão vai saindo de baixo dele e ele nunca o alcança. A velocidade para que isso ocorra, a cada altura, é determinada igualando a força centrípeta necessária para que fique circulando a Terra naquela altura, mv²/(R+h), com a força fornecida pela gravidade ali, GMm/(R+h)², em que m é a massa do corpo, v é sua velocidade, M é a massa da Terra, R é o raio da Terra, h é a altura e G é a constante da gravitação. Igualando, se tira que a velocidade tem que valer: v = (GM/(R+h))^(1/2). Isso tem que ser fornecido pelo foguete que vai lançar o satélite ou nave em órbita na altura h. Se a altura for zero, a fórmula dá a velocidade que um corpo teria que ter para ficar em órbita rasante ao solo (pelo menos acima da mais alta montanha). Todavia, como se está dentro da atmosfera, há o atrito do ar, que iria diminuir essa velocidade, sendo, pois, necessária, uma propulsão constante para compensar. Note que a velocidade orbital vai diminuindo à medida que a altura aumenta. Fazendo os cálculos se acha que a velocidade da órbita rasante é cerca de 37.000 km/h. Ou seja, se um avião tiver essa velocidade, ele estará em órbita.

Por que existe algo ao invés de nada?

Não há razão nenhuma para isso. Poderia não existir nada mesmo. Se há algo, é por mero acaso. Todavia recomendo a leitura de uma boa discussão sobre o assunto:
Por que existe algo em vez do nada? - Jim Holt - Intrínseca.

Como é que a beleza pode salvar o mundo, se exigir vítimas?

Mas a beleza não tem que exigir vítimas. Por outro lado, mesmo assim, não salva o mundo. A bondade, sim, pode salvar o mundo, se for acompanhada da diligência, do compromisso e do empenho, bem como da sinceridade, da honestidade e da justiça. Pela bondade se entende o compartilhamento, a solidariedade, a colaboração, a generosidade e atitudes desse tipo. Que não têm nada a ver com a beleza.

Professor, alguns países democratizaram o acesso ao ensino superior. Não acha que no Brasil o vestibular é excludente? E uma curiosidade: li que o vestibular foi adotado durante a ditadura militar nos anos 60, como dava-se o ingresso antes?

O que a ditadura fez foi criar os vestibulares unificados para todos os cursos da mesma universidade e os que abrangiam várias universidades, como a Fuvest e o Cesgranrio. Antes a admissão aos cursos era feita por uma seleção diferente para cada um, mesmo que da mesma universidade. Em geral as provas eram todas abertas e em dias diferentes. A questão da democratização do acesso esbarra em dois problemas que são ligados. Para começar, no Brasil todos pretende fazer curso superior. Em outros países, mesmo desenvolvidos, muitos ficam no Ensino Médio Profissionalizante, que já propicia um bom rendimento. Depois, em razão disso, não há vagas suficientes para atender a demanda. Todavia, mesmo no primeiro mundo, o acesso ao nível superior é eletivo. Na França isso é feito pelo "baccalauréat". Nos Estados Unidos há cotas para as escolas, em que a seleção é feita pelas notas dos alunos. A seleção lá é subjetiva e inclui outros fatores também.

Como estudar e saber tanto assim igual o senhor? Eu leio, leio, mas nada entra na minha cabeça, quando termino esqueço tudo

Isso é uma característica mental da pessoa. Não sei porque eu guardo tudo o que estudo. Não faço esforço nenhum. Todavia há técnicas de memorização, que podem ser aprendidas em livros específicos. Nunca as pratiquei, contudo.

erwin schrödinger desenvolveu sua famosa equação através de um raciocínio muito bem urdido?

Muito mesmo. Ele se baseou nas relações de De Broglie, na proposta de Bohr para as ondas estacionárias de elétrons e, por analogia com as equações de Maxwell, induziu sua famosa equação para a função de onda.

João 1:18 vs Êxodo 24:9 - 11 Ora, se JHVH foi visto por todas aquelas pessoas, incluindo os setenta anciãos de Israel, e o texto de João diz que nunca homem nenhum viu a Deus, o que pensa sobre essas contradições de autores da bíblia?

Penso, simplesmente, que a Bíblia é uma obra de ficção, no que diz respeito a suas assertivas sobre os hipotéticos entes de alguma realidade sobrenatural, a par de um livro sobre as práticas litúrgicas judaicas e um tratado de moral, parte esta que é aproveitável em muitos pontos (mas não todos).

https://ask.fm/wolfedler/answers/141494847005 você havia respondi ele

Em relação à existência de deuses, pode-se ser:
Ateísta
Monoteísta
Politeísta
Monodeísta
Polideísta
Panteísta
Panenteísta
Pandeísta
Panendeísta
Henoteísta
Em relação ao modo como se encaram essas concepções, pode-se ser:
Gnóstico
Agnóstico
Ignóstico
Cético
Conjugando os dois aspectos se tem quarenta possibilidades.

Quanto tempo levará para que os humanos dominemos por completo a tecnologia de trasnmutar a matéria?

A questão é que a transmutação se dá átomo por átomo e não é controlada. Se uma amostra for radioativa ela vai se transmutar naturalmente. Dependendo da meia vida do isótopo isso pode levar milhares de anos. Ainda não se sabe como provocar o decaimento a uma taxa maior do que a natural. Nem sei se isso seria possível.

Ernesto, existe transmutação da matéria por intermédio da física? Estava a ler algo sobre Rutherford, que me deixou intrigado. É possível trasmutar nitrogénio em oxigénio por bombardeio de partículas alfas?

Certamente que sim. Decaimentos radioativos e absorção de radiação podem mudar um elemento em outro. Assim é que se faz, por exemplo, datação geológica de rochas e fósseis. Todavia, como procedimento para a obtenção de minerais raros, não é economicamente viável.

Olá, professor! Eu penso que devido a ciência ser materialista e com o pensamento mais secularista, naturalmente ela é mais progressista, por isso vai contra muitas ideias conservadoras religiosas, e não somente doutrinação escolar como dizem, o que acha? Gracias!

Isso mesmo. A ciência não considera nenhuma assertiva religiosa, seja de que religião for. Cientistas, em particular, podem possuir alguma crença religiosa, mas isso não pode interferir em seu trabalho científico. Porque a ciência busca a verdade, como revelada pelos fatos, de modo verificável e justificado. Daí ela ser fisicalista, ou seja, considera que a realidade objetiva (isto é, exterior às mentes), não possua elementos sobrenaturais. Todavia não foge à ética, como norteadora das ações. Mas a ética não tem nada a ver com as crenças religiosas. O secularismo da ciência significa que ela considera que, também, deva se voltar para a promoção do bem do mundo, de modo desvinculado a qualquer noção mística religiosa, mas apenas pelo valor intrínseco do bem como provedor de felicidade, alegria, prazer, bem estar, satisfação e tudo de bom para o maior número de seres, inclusive não humanos. Isso é algo que a escola precisa incutir na infância e na juventude como um valor superior a ser buscado, ou seja, a verdade, esteja ela de acordo ou desacordo com as crenças religiosas, sejam quais forem. Isso não é uma doutrinação e sim uma conscientização de valores corretos e de responsabilidades a serem assumidas, de modo inteiramente esclarecido e livre.

Qualquer pessoa com força de vontade e inteligência razoável pode se tornar muito inteligente. Em dois anos, estudando todos os dias, ao final são 730 dias de estudo equilibrado.

A inteligência, certamente, pode ser aprimorada por meio de um treinamento. Isso é do que cuida a neuróbica. Todavia o acréscimo dificilmente ultrapassa um quinto, no máximo um quarto, do quanto já se tem de inteligência. Assim, para se ficar muito inteligente, é preciso que já se seja bem inteligente. Por exemplo, para chegar a um QI de 160, há que se partir de um QI de uns 130. Esse ponto de partida é o que se tem geneticamente acrescido do que se aprimorou na tenra infância, que é a época de melhor aproveitamento dos processos de aperfeiçoamento da inteligência. Note que grande inteligência não significa domínio de vastos conhecimentos. Uma pessoa pode ter um imenso cabedal de conhecimentos e não ser muito inteligente, do mesmo modo que pode ser extremamente inteligente e não dominar muitos conhecimentos. A inteligência se liga, principalmente, à habilidade de adquirir conhecimentos e habilidades sem muito esforço e solucionar, também sem muito esforço, problemas de qualquer ordem, inclusive da vida pessoal.

o matemático john lennox disse que "provas" só existem no campo que ele atua, a matemática. Não há como falar em provas em ciência, mas em evidências. Isso seria devido ao caráter conceitual de "prova", que é uma demonstração lógica por meio de raciocínio?

Em Física, pelo menos, além das leis, que são empíricas, isto é, verificadas por evidências experimentais ou observacionais, também há os teoremas, que são deduzidos delas por demonstrações lógicas, normalmente matemáticas. Sua veracidade decorre da veracidade das leis, que funcionam como os postulados na Matemática. Além disso, há, também, as definições, que são arbitrárias e os princípios, que são induzidos e cuja veracidade decorre da veracidade das leis que deles podem ser deduzidas. Uma demonstração ou prova lógica, contudo, não necessariamente tem que ser matemática. Pode ser um raciocínio lógico conduzido por meio de juízos expressos em proposições.

Professor, na sua opinião, qual é melhor para avaliar a aptidão do aluno a um curso superior, um vestibular com múltiplas escolhas ou um com questões dissertativas?

As questões dissertativas são preferíveis, contudo, são inviáveis devido ao grande tempo que sua correção exige. Então se recorre às de múltipla escolha, que podem ser corrigidas por processos automáticos. Quando bem formuladas, as questões de múltipla escolha permite uma avaliação boa também. Especialmente se se usar a metodologia da Teoria de Resposta ao Item, aplicada no ENEM. Também é bom que se faça uso de questões com maior número de opções (oito, por exemplo) e com a possibilidade de haver mais de uma correta, o que permite a atribuição de acerto parcial a cada uma. Inclusive com a atribuição de fração de ponto negativa às marcações incorretas.

estava procurando em alguns sites as disciplinas mais difíceis que existem. dentre elas achei a matemática, o direito, a anatomia, mas sempre em primeiro, estava a a física quântica ou simplesmente física. Você concorda? Por que física quântica é tão difícil?

Por duas razões. A primeira é que ela não é intuitiva, isto é, seus conceitos não correspondem ao que se percebe diretamente do mundo. Ou seja, é altamente abstrata. A segunda é que, para modelar a estrutura e o funcionamento dos sistemas moleculares, atômicos e subatômicos se requer um ferramental matemático bem sofisticado, que não é disponibilizado nem nos cursos básicos de cálculo diferencial e integral, álgebra linear e geometria analítica do ciclo básico dos cursos superiores de ciências exatas. Para isso os cursos de Física oferecem a disciplina "Métodos Matemáticos da Física". Como, em geral, grande parte das pessoas têm uma formação incipiente em Matemática, elas não conseguem absorver o conteúdo da Física Quântica, da Física de Partículas Elementares, das Teorias Clássica e Quântica de Campos, da Relatividade Restrita e Geral e da Cosmologia. A Física elementar, dada no Ensino Médio, não apresenta dificuldade. Quem não a domina é porque tem uma falta de base em Matemática, desde o nível fundamental. Alguns, todavia, apresentam uma dificuldade de assimilação conceitual mesmo da Física. A questão é que a Física, mesmo a elementar, requer um bom traquejo de abstração e raciocínio lógico, mesmo que não matemático. Muitas pessoas não conseguem se elevar acima do concreto como, por exemplo, raciocinar hipoteticamente, inclusive em temas que não têm nada a ver com Física. Isso é decorrente de uma falha no processo educativo no nível fundamental, que privilegia a aquisição de conhecimentos em detrimento do desenvolvimento de habilidades e do aprimoramento da inteligência. Para resolver isso é preciso, antes de mais nada, remunerar MUITO BEM, o magistério de forma a fazer a profissão economicamente atrativa para as melhores cabeças, que vão preferir o magistério à medicina, à engenharia, ao direito e outras profissões de estatuto mais elevado pela razão de que o magistério provê melhores rendimentos do que elas.

Acha que é verdade ou mentira que o emocional pode atrapalhar provas, concursos e até mesmo provas acadêmicas?

É verdade.

Eu passei direto do 3o ano pra faculdade, porém considero meu 3o ano uma confusão, vejo pessoas em minha volta que se esforçaram mais que eu pra estar aqui, e parece que sou um caso à parte. Devia eu ter estudado mais ou minhas capacidades me trouxeram aqui de forma justa?

Se você não trapaceou na feitura dos exames admissionais, então você entrou de modo justo. O que importa não é o quanto se estudou e sim o quanto se aprendeu. Há quem seja capaz de aprender sem estudar. Outros, mesmo estudando, não aprendem. A diferença é a inteligência. Não é mérito pessoal ser inteligente mas, se se é, é justo que tire proveito disso, exceto para fazer o mal.

A força normal é mais forte que a gravidade? É ela que não nos deixa afundar no chão?

Depende. A força de gravidade é proporcional à massa enquanto a força normal se deve à interação entre as partículas de um sólido. Dependendo das massas envolvidas, a gravidade pode ser maior ou menor do que a normal. Mas a normal é sempre uma reação. Ou seja, ela não toma a iniciativa de ser exercida, mas só responde às solicitações. Todavia se a solicitação for grande ela pode não ser capaz de responder, levando à ruptura do material. Estenda uma folha de papel presa pelas bordas e coloque um peso sobre ela. Até certo valor dele, o papel o suporta. Além de certo valor, o papel rasga. No caso do apoio do solo, a normal pode não suportar alguma solicitação também, como no caso dos bate-estacas, que a fincam no solo.

Não é conveniente a ninguem,ficar horas sem se alimentar,especialmente quando se é diabético,pois pode ocasionar uma hipoglicemia.Alem do mais, o estômago que está ininterruptamente fabricando ácido clorídrico para digerir alimentos, se não os recebe ocasiona uma gastrite.

Quando eu fico sem almoçar eu me alimento fazendo lanche. Só não como comida. Mas isso é raro. Só que não faço questão de ter que almoçar. Não tenho hora de comer. Como quando a fome bate. Durmo quando o sono chega. Acordo quando o sono acaba.

O que pensa das competições de Física,Matemática e outras ciências?

Não gosto desse tipo de coisa. Não gosto de competições de espécie nenhuma. Especialmente esportivas. Acho que tudo deve ser feito sem pretender se ser melhor do que ninguém. Sem querer vencer. A não ser superar a si mesmo. Pelo contrário. Todo mundo ajudando uns aos outros para que todos sejam excelentes. Sem competição. Só com colaboração. Pode haver atividade esportiva sem disputa. Da mesma forma que pode haver atividade do tipo das olimpíadas de ciências, em que as pessoas são desafiadas a superarem-se a si mesmas, sem disputa. Não acho que qualquer tipo de disputa, competição e concorrência seja algo desejável para o bem do mundo.

Qual veio a existir primeiro, a luz ou as trevas? A luz precisa de um corpo para que possa alumiar, todavia, uma vez que a energia desse corpo se esgota, esta logo se apaga. Sendo assim, as trevas podem durar uma eternidade, mas a luz não dura para sempre?

No surgimento do Universo havia só campo, sem matéria e nem radiação. Como luz é radiação, podia-se dizer, então, que tudo era treva. A matéria surgiu antes da radiação, pois esta é produzida por aquela. No princípio o Universo era inteiramente opaco, como o interior de uma única estrela. Portanto, inteiramente negro. Só com o desacoplamento da matéria e da radiação, que ocorreu 380 mil anos após o Big Bang, é que o Universo se tornou transparente e a luz pode se propagar. Mas, então, ela era quase exclusivamente formada por radiação gama. A radiação visível só apareceu com o surgimento das estrelas, uns 550 milhões de anos depois do Big Bang. Com a expansão do Universo, em algum momento, todas as estrelas se apagarão e toda a radiação já lançada no espaço atingirá comprimentos de onda superiores ao da luz visível, voltando, portanto, tudo a ser inteiramente negro. Ou seja, as trevas sobreviverão à luz.

O que é um dipolo elétrico? E por que não pode existir monopolos elétricos e magnéticos?

Um dipolo é um sistema que apresente dois polos opostos dentro dele mesmo, em relação ao fenômeno considerado. No caso elétrico ele possui uma região positivamente carregada e outra negativamente, com o mesmo módulo da carga, só que de sinal contrário. No caso magnético ele possui duas regiões, uma das quais o campo magnético sai do sistema para seu exterior, denominada pólo norte e outra da qual o campo magnético passa do exterior para o interior do sistema, denominado seu pólo sul. Como o campo magnético sempre se apresenta em linhas fechadas, é preciso que haja um lugar em que elas entrem e outro em que elas saiam, não podendo só entrar ou só sair de um sistema. Já o campo elétrico pode só sair de um sistema, se ele estiver positivamente carregado ou só entrar, se ele estiver negativamente carregado. Então é possível haver monopolos elétricos, mas não magnéticos. A menor estrutura magnética é sempre um dipolo, mas a elétrica é um monopolo, Algumas teorias aventam a possibilidade da existência de monopolos magnéticos, mas isso nunca foi observado. Matematicamente isso se exprime pelo fato do campo magnético possuir sempre divergente nulo e o elétrico poder possuir divergente não nulo, que é proporcional à densidade de carga elétrica do entorno do ponto considerado. Isso é estatuído pela chamada "Lei de Gauss" e representa um comportamento observado na natureza.

Então quer dizer que existe força normal atrativa?

Pode haver. Normal é a componente da força de contato perpendicular às superfícies de contato. A paralela é o atrito. Se se tiver, por exemplo, duas superfícies de clivagem de dois cristais, no contato eles podem se atrair, quando, então, a normal é chamada de adesão. Em geral a normal é repulsiva. A força que faz uma lagartixa andar pela parede é uma força de adesão, provocada por interações de Van der Walls. No caso de insetos é diferente. Veja isto:
http://mundoestranho.abril.com.br/mundo-animal/como-alguns-animais-conseguem-subir-nas-paredes/

É o budismo uma religião ateia?

Sim. Mas acredita em alma e espíritos, bem como em reencarnação.

Professor, você, quando pode, dorme durante o dia? Se sim, desde que idade faz isso?

Sistematicamente, não. Mas, se estou muito cansado ou se dormi pouco na noite anterior, posso dormir de dia sim. Sempre fiz isso, pois nunca fui regular nos horários de dormir e acordar. Tem dia que durmo só quatro horas, tem dia que durmo dez. Do mesmo modo que não sou regular na hora de tomar refeições. Normalmente almoço à uma da tarde, mas posso tanto almoçar às onze quanto às três da tarde. Ou não almoçar.

Existe alguma relação entre a gravidada, a força eletromagnética que não permite que atravessemos o chão quando caímos e a força de van der waals?

A gravidade não tem nada a ver com o eletromagnetismo, mas a interação de Van der Waals sim. Esta, como as pontes de hidrogênio, se devem a atrações devido ao surgimento de dipolos elétricos em átomos ou moléculas que não chegam ocasionar a fusão dos orbitais e nem levam à captura de elétrons de um átomo pelo outro. A força repulsiva que impede a interpenetração dos corpos sólidos provém do fato de seus átomos, íons ou moléculas estarem fortemente ligados de modo que a aproximação de outro corpo provoca, entre as camadas superficiais dos elétrons de ambos, uma repulsão que dificulta (mas não impede em definitivo) que a coesão interna entre as moléculas, átomos ou íons dos dois seja rompida e haja a ruptura de estrutura. Todavia, se a pressão for grande, como acontece com o gume de uma faca, isso pode acontecer.

Para você, o que é a religião? Ser ateu, significa não ter religião? A religião, para ser considerada como tal, necessita da crença em deus(es)?

Toda pessoa ateia não tem religião, mas uma pessoa pode não ter religião e não ser ateia. E pode ter uma religião e ser ateia. Religião é uma instituição baseada em um conjunto de crenças relativas ao significado da vida, ao comportamento ético, à existência de entidades espirituais, ao destino da alma e, possível, mas não necessariamente, à existência de divindades. Pessoas podem ter crenças relativas e tais assuntos sem ser filiada a religião nenhuma.

Eugênio. Gostaria de ouvir tua opinião sobre traição. O caso é que sou comprometido há 4 anos com uma garota fantástica, tudo que um homem pode querer. Porém ela foi minha primeira namorada. E eu sinto atração por outras mulheres, mas não quero trai-la por medo de perdê-la. O que faço?

Traição é inaceitável pelo fato de envolver uma mentira. Mas não é preciso se ter exclusividade nos relacionamentos amorosos, desde que isso seja sabido e consentido pelos envolvidos. Converse com sua companheira e estabeleçam um pacto de não exclusividade, em que continuam a relação mas com a liberdade de manterem, também, ambos, outros relacionamentos paralelos. Se ela não concordar você tem que optar ou por ficar só com ela ou ficar sem ela e com outros relacionamentos. Todavia é bom pensar antes de falar, porque a probabilidade de que ela não concorde é maior. E, então, mesmo que você opte por ficar com ela, pode ser que ela não queira mais.

o que você acha da teoria de que a Lua é um pedaço da Terra?

Bem plausível. Segundo ela a Terra teria se chocado com outro planeta que ser formara na mesma órbita, chamado Theia e, nesse choque, parte dele se incorporou à Terra e parte, juntamente com parte da Terra arrancada, formou a Lua.

Se escovar os dentes fosse suficiente, o homem teria chegado a lua?

Escovar os dentes não é suficiente e o homem, de fato, foi à Lua.

Tens interesse em gnosticismo? Já leste algo sobre o demiurgo? O que pensas a respeito? A igreja católica perseguiu muitos gnósticos no passado.

Gnosticismo é algo tão inverossímil quanto qualquer concepção religiosa, pois admite que exista algum tipo de realidade transcendental, espiritual, divina ou o que seja de caráter sobrenatural. O mundo objetivo, isto é, daquilo que existe por si mesmo e não seja uma concepção puramente mental, como as abstrações, é puramente físico, isto é, composto de campo, radiação e matéria. Não existem entidades objetivas não físicas. Ou seja, tudo que pode existir sem ser uma concepção puramente mental, como as instituições, as normas e leis, as figuras geométricas, os números e o que for do tipo, tem que ser, necessariamente, físico, ou seja, feito de campo, radiação ou matéria (mas não de energia, como dizem alguns, pois energia não é um constituinte de algo e sem uma propriedade de sistemas).

Quem gostarias de contactar numa sessão espírita?

Isso é uma completa baboseira. Espíritos não existem. Portanto não se encarnam e nem se comunicam com ninguém. A vida é só um fato biológico e se encerra com a morte.

Professor em uma postagem recente sua você disse que o aquecimento global é de 2 naturezas, uma delas você diz que não é antrópico, você poderia citar as fontes deste argumento?

Tem uma revista de meteorologia que dedicou um número especial ao assunto. Vou procurar para te passar a referência. Li tudo e, então, fiquei sabendo bem a respeito. Há, inclusive, fraudes do IPCC. É bom ver o outro lado da questão, para tomar ciência e formar uma opinião:
http://www.scielo.br/pdf/ss/v13n3/2316-8994-ss-13-03-00643.pdf
http://scienceblogs.com.br/curupira/2012/12/vazamento-de-relatorio-ipcc-confirma-fraude-aquecimento-global-so-nao/

Já leste Admirável Mundo Novo? Curtes esse tipo de ficção científica? O que pensas sobre Aldous Huxley?

Sim, há várias décadas. Gosto do Huxley. Também li "De volta ao Admirável Novo Mundo". Aliás, adoro ficção científica e romances policiais. Já li todos os do Asimov e todos os da Aghata Christie, bem como do Conan Doyle. Além de muitos outros de ficção científica, como a série toda de Duna e de Shikasta. Atualmente tenho lido mais divulgação científica, livros sobre Ateísmo e Filosofia. Agora, por exemplo, estou lendo dois: "The Life of the Cosmos" do Lee Smolin e "História da Literatura Ocidental" de Sandra Newman. Também tenho o do Otto Maria Carpeux, mas são quase quatro mil páginas e ainda não animei a ler.

Professor, você acredita que viverá, pelo menos, mais uns 30 anos?

Não. Tenho insuficiência cardíaca. Já tive dois infartos, um em 2001 e um em 2002. Em 2015 coloque três stents nas coronárias em uma angioplastia. Acho que não vivo mais uns dez anos, se viver. Além disso, sou diabético.

Você acha que compensa revisar todo conteúdo de exatas do ensino médio estando cursando o ensino superior? Quanto tempo despenderia? É andar pra trás?

Se você fez um bom ensino médio, não precisa revisar tudo. Basta ir vendo o assunto que for precisando, à medida que for aparecendo. Mas é bom ter os livros de matemática e física do ensino médio à mão, para consulta. Existem, também, uns livros chamados de "Pré-Cálculo", muito interessantes para dar uma base necessária para o cálculo, a álgebra linear e a geometria analítica. Outros bons, como suplementares, são os da "Coleção Schaum".:
https://www.google.com.br/search?q=cole%C3%A7%C3%A3o+Schaum&source=lnms&tbm=isch&sa=X&ved=0ahUKEwiRxs3e9aXVAhXBDJAKHUjDCQ4Q_AUICygC&biw=1600&bih=770
Também é importante se ter à mão o "Manual de Fórmulas e Tabelas Matemáticas":
https://www.google.com.br/search?q=manual+de+f%C3%B3rmulas+e+tabelas+matem%C3%A1ticas&source=lnms&tbm=isch&sa=X&ved=0ahUKEwjLuLSj9qXVAhVCj5AKHUZ_AFkQ_AUICigB&biw=1600&bih=770

Prefere, particularmente, o mundo do século 20 ou do 21?

Por enquanto estou preferindo o século 21. Mas é cedo para se fazer uma apreciação dele. Vamos esperar 83 anos.

Crescer num lar restritivo e violento, onde a criança foi até negligenciada algumas vezes, aumenta muito as chances dessa criança se tornar um jovem/adulto pouco racional e até mesmo ser "burro"? Ou isso seria mais uma desculpa usada para fugir de estudos e práticas para desenvolver a inteligência?

Na tenra infância (até uns quatro anos), a rejeição afetiva pode sim, comprometer o desenvolvimento da inteligência. Depois, contudo, nem tanto. Pode sim, haver uma desculpa para não se esforçar, por preguiça, alegando esse tipo de problema. Mas pode ser que seja verdade mesmo. Só um teste psicológico poderá diagnosticar a verdadeira razão: falta de inteligência ou preguiça.

O que fazer quando se ama uma pessoa e ela não se importa com voce?

Amá-la apenas platonicamente e, paralelamente, se abrir para amar, também, a outra pessoa que, possivelmente, lhe corresponda. Mas não precisa deixar de amar a primeira. Todo amor é bom, mesmo que não seja correspondido. Só que a segunda tem que saber que você ama também a primeira. E saber que você a ama igualmente com a mesma intensidade e sinceridade. Amor não precisa e não tem que ser exclusivo. Você também tem que admitir que a pessoa que o ama possa, também, amar a outrem. Não só amar mas se relacionar amorosamente, tanto romântica quanto eroticamente. O que não pode haver é traição, isto é, manter relacionamentos paralelos ocultos de uma das pessoas (ou das duas ou mais). Se for tudo conhecido e consentido não há traição, apenas não exclusividade.

Professor eu notei alguns detalhes nesse movimento anti-ciência e um deles eh o seguinte muitos desses movimentos são liderados por fanáticos religiosos e um dos principais eh o movimento anti-vacina pois defendem que a vacina faz mal a saúde e que da autismo o que fazer quanto a esses imbecis?

É mais ou menos como os Testemunhas de Jeová, que não aceitam transfusão de sangue. Acho que esse pessoal, quando ficar doente, não pode chamar médico e nem ir para o hospital. Tem que ficar só rezando para Deus curar. Só que não pode impingir isso a quem não tenha discernimento e nem poder de decisão, como seus filhos crianças. A solução é a educação escolar, desde os mais tenros anos, ir tirando essas idéias estúpidas da cabeça da criançada para que elas ensinem a seus pais. Tem que mostrar, sem pruridos, como o criacionismo é uma baboseira e como a evolução é que é certa mesmo. Não se pode explicar a realidade por meio de crenças.

Uma boa memória ajuda a ir bem no ensino superior?

Certamente que sim. Uma boa memória é um dos aspectos da inteligência, mesmo que não seja o único nem o mais importante. Mas é muito importante. Todavia tem que ser a memória conceitual e relacional, bem como a procedural e não apenas a memória das coisas e dos fatos. O importante, para o aprendizado, é a memória dos significados, das implicações e dos procedimentos.

LinkWithin

Related Posts with Thumbnails